Сохранен 509
https://2ch.hk/sci/res/308111.html
24 декабря Архивач восстановлен после серьёзной аварии. К сожалению, значительная часть сохранённых изображений и видео была потеряна. Подробности случившегося. Мы призываем всех неравнодушных помочь нам с восстановлением утраченного контента!

Математика, тред 33

 Аноним 30/10/15 Птн 16:56:05 #1 №308111 
14462133650400.jpg
Начну этот тред с трех задач.
1. Верно ли, что в каждом конечном графе имеются вершины одинаковой степени?
2. Верно ли, что если в конечной группе G выбрано подмножество A такое, что каждый g из G сопряжен некоторому a из A и элементы A попарно коммутируют, то G коммутативна?
3. Пусть A подмножество вещественной прямой. Верно ли, что не бывает такого, чтобы одновременно A имело нулевую меру и дополнение A было бы объединением счетного числа нигде не плотных?
Аноним 30/10/15 Птн 17:06:37 #2 №308116 
32. >>306161 (OP)
31. >>304279 (OP)

21. http://arhivach.org/thread/93067/
22. https://arhivach.org/thread/94240/
23. https://arhivach.org/thread/95680/
24. https://arhivach.org/thread/96720/
25. https://arhivach.org/thread/99481/
26. https://arhivach.org/thread/100880/
27. https://arhivach.org/thread/101335/
28. http://arhivach.org/thread/106743/
29. http://arhivach.org/thread/109198/
30. http://arhivach.org/thread/114111/
sage 30/10/15 Птн 17:14:50 #3 №308117 
Зачем плодить одинаковые треды? Ты даун?
Аноним 30/10/15 Птн 17:18:55 #4 №308121 
>>308117
Шапка того треда просто ужасна.
sageАноним 30/10/15 Птн 17:24:11 #5 №308123 
>>308121
Иди в тимфортрес поиграй, шапочник
knuebok 30/10/15 Птн 18:05:40 #6 №308126 
>>308123
На самом деле рли отсутствие ссылок на архивач - очень-очень плохо, хоть я ими никогда и не пользуюсь, но просто сама возможность проскроллить любой когда-либо существоваший тред греет душу :3
knuebok 30/10/15 Птн 18:10:20 #7 №308127 
>3. Пусть A подмножество вещественной прямой. Верно ли, что не бывает такого, чтобы одновременно A имело нулевую меру и дополнение A было бы объединением счетного числа нигде не плотных?

Известный очень сюжет, я когда-то слышал название этому как: "теорема о колбасе: колбасу (отрезок [0..1]) можно разрезать на две части - невидимую (первой категории) и невесомую (меры 0)". Не думаю, что до той конструкции, что я знаю реально самому додуматься, но палить контору не буду, кто захочет - нагуглит.
Аноним 30/10/15 Птн 18:23:27 #8 №308130 
>>308127
>теорема о колбасе
Ох, как же я не люблю этот советский стиль юмора.
Аноним 30/10/15 Птн 18:23:37 #9 №308131 
>>308127
Справедливости ради, я самостоятельно эту задачку никогда не решал. Но смотря на известную мне конструкцию, мне кажется, что придумать этот контрпример можно просто здраво проанализировав определения этих двух понятий "быть маленьким множеством".
Аноним 30/10/15 Птн 19:09:42 #10 №308148 
>>308111 (OP)
>1. Верно ли, что в каждом конечном графе имеются вершины одинаковой степени?
Да, степень каждой вершины не превышает n-1, где n число вершин графа. Следовательно согласно принципу Дирихле существуют минимум две вершины с равной степенью.
Примечание: необходимо чтобы граф был простым.
Аноним 30/10/15 Птн 19:14:48 #11 №308151 
>>308148
А ничего, что степени верши - числа от 0 до n-1 (т.е. n различных возможностей).
Аноним 30/10/15 Птн 19:16:24 #12 №308153 
>>308151
Контрпример.
Аноним 30/10/15 Птн 19:19:29 #13 №308154 
>>308153
Я не говорю, что заключение неверно, а указываю на дырку в доказательстве.
Аноним 30/10/15 Птн 19:26:37 #14 №308155 
>>308154
Каким образом факт, что степени вершин – числа от 0 до n-1 показывает на дыру в доказательстве? Возможностей кстати не n.
Аноним 30/10/15 Птн 19:33:38 #15 №308159 
>>308155
>Следовательно согласно принципу Дирихле существуют минимум две вершины с равной степенью.
Чтобы воспользоваться здесь принципом Дирихле нужно, чтобы возможных значений степеней вершин было бы меньше, чем самих вершин.
>Возможностей кстати не n.
Ну так докажи это и будет решение.
Аноним 30/10/15 Птн 19:40:25 #16 №308165 
>>308159
>Чтобы воспользоваться здесь принципом Дирихле нужно, чтобы возможных значений степеней вершин было бы меньше, чем самих вершин.
Вершин n. Степень каждой вершины v: d(v) ≤ n-1
Аноним 30/10/15 Птн 19:44:06 #17 №308171 
>>308165
>Степень каждой вершины v: d(v) ≤ n-1
Да. И это n возможных значений. Например, в графах на 4 вершинах могут встречаться вершины степени 0, 1, 2 и 3.
Аноним 30/10/15 Птн 20:08:47 #18 №308201 
>>308171
Верно, благодарю за замечание.

>Да, степень каждой вершины не превышает n-1, где n число
Предположим, что для любого v: 1 ≤ d(v) ≤ n-1 тогда утверждение доказано.
Иначе, существует вершина u т.ч. d(u) = 0.
Если она не единственная - утверждение доказано.
Иначе исключаем u и получаем n-1 вершин v, где 1 ≤ d(v) ≤ n-2 – согласно приниципу Дирихле существуют минимум две вершины с равной степенью. Утверждение доказано.
Аноним 30/10/15 Птн 20:15:31 #19 №308205 
Берем вершину с самой большой степенью - предположим она равна N. Так как граф простой (для непростого графа утверждение неверно) то это вершина соединена с N другими вершинами. Так как вершин N+1 штука, а их степень может принимать значения от 1 до N - то как минимум две вершины имеют одинаковую степень.
Аноним 30/10/15 Птн 20:18:09 #20 №308207 
>>308201
Да, теперь в самом деле решение.
Аноним 30/10/15 Птн 20:27:01 #21 №308211 
>>308205
Изящнее.
Аноним 30/10/15 Птн 23:02:09 #22 №308259 
Рассмотрим граф из n вершин. Если в графе есть две или более вершины степени 0, то все. Если такая вершина одна, имеем n - 1 вершин со степенью от 1 до n - 2. Если таких вершин нет, имеем n вершин со степенью от 1 до n - 1.
Аноним 30/10/15 Птн 23:08:40 #23 №308262 
>>308111 (OP)
Че такое сопряжен, не напомнишь?
Хоть и не знаю, но полагаю, что решение 2й такое: g a g g b g = g a b g = g b a g = g b g g a g
Аноним 30/10/15 Птн 23:52:33 #24 №308267 
>>308262
Элемент $a$ сопряжен элементу $b$, если $a=gbg^{-1}$ для некоторого $g$.
Аноним 31/10/15 Суб 00:14:30 #25 №308272 
>>308126
1. https://arhivach.org/thread/18638/
2. https://arhivach.org/thread/27246/
3. https://arhivach.org/thread/27696/
4. https://arhivach.org/thread/38709/
5. https://arhivach.org/thread/46502/
6. https://arhivach.org/thread/48852/
7. https://arhivach.org/thread/52165/
8. https://arhivach.org/thread/56479/
9. https://arhivach.org/thread/63306/
10. https://arhivach.org/thread/70618/
11. https://arhivach.org/thread/74342/
12. https://arhivach.org/thread/74341/
13. https://arhivach.org/thread/76561/
14. https://arhivach.org/thread/78408/
15. https://arhivach.org/thread/79152/
16. https://arhivach.org/thread/82499/
17. https://arhivach.org/thread/92427/
18. https://arhivach.org/thread/84722/
19. https://arhivach.org/thread/87923/
20. https://arhivach.org/thread/91329/
21. http://arhivach.org/thread/93067/
22. https://arhivach.org/thread/94240/
23. https://arhivach.org/thread/95680/
24. https://arhivach.org/thread/96720/
25. https://arhivach.org/thread/99481/
26. https://arhivach.org/thread/100880/
27. https://arhivach.org/thread/101335/
28. http://arhivach.org/thread/106743/
29. http://arhivach.org/thread/109198/
30. http://arhivach.org/thread/114111/
31. https://arhivach.org/thread/116099/
32. https://arhivach.org/thread/118093/
Аноним 31/10/15 Суб 01:01:59 #26 №308276 
Кто-нибудь ходит сюда?
http://ium.mccme.ru/f15/f15-krasilschik.html

На странице
http://www.mathnet.ru/php/conference.phtml?eventID=25&confid=763&option_lang=rus&if_videolibrary=1
последние две лекции отсутствуют, кому-нибудь известно, что что там было? и где можно посмотреть
Аноним 31/10/15 Суб 03:52:18 #27 №308287 
>>308111 (OP)
>2. Верно ли, что если в конечной группе G выбрано подмножество A такое, что каждый g из G сопряжен некоторому a из A и элементы A попарно коммутируют, то G коммутативна?
Нет. Контрпример группа симметрий D_4:
D4 = {id, r1, r2, r3, fv, fh, fd, fc}
A = {id, r1}
Каждый элемент из D4 сопряжен с id из A,
id, r1 коммутируют, но D4 некоммутативна т.к. (например) r1 fv ≠ fvr1.

Аноним 31/10/15 Суб 11:08:50 #28 №308315 
>>308287
Единичный элемент не сопряжен никому кроме самого себя (в любой группе).
Аноним 31/10/15 Суб 12:04:33 #29 №308324 
>>308303
Да. Можно строить быстрорастущие функции из счетных ординалов с использование "быстрорастущей" иерархии https://en.wikipedia.org/wiki/Fast-growing_hierarchy . Стоит отметить, что скорость роста функции Аккермана достигается уже на $f_\omega(x)$ ($\omega$ наименьший бесконечный ординал, он соответствует порядковому типу натуральных чисел).
Приведенные выше функции, при некотором ограничение на ординалы, оказываются вычислимыми. Есть растущие еще быстрее, но уже невычислимые функции, например функция сопоставляющая натуральному числу $n$ наибольшее число с колмогоровской сложностью не превосходящей $n$.
Аноним 31/10/15 Суб 13:00:13 #30 №308329 
>>308315
Он в A, а значит условие
>каждый g из G сопряжен некоторому a из A
Выполняется.
Аноним 31/10/15 Суб 13:04:47 #31 №308331 
>>308329
Проблема в том, что это утверждение неверно
>Каждый элемент из D4 сопряжен с id из A,
Аноним 31/10/15 Суб 14:08:06 #32 №308339 
14462896865070.jpg
Семинары в москве есть какие?
Аноним 31/10/15 Суб 14:27:07 #33 №308353 
>>308339
В Москве огромная куча математических семинаров. Тебя что конкретно интересует?
Аноним 31/10/15 Суб 14:38:00 #34 №308359 
>>308329
>Он в A
Не факт. A - это подмножество центра, а не центр.
Аноним 31/10/15 Суб 15:05:21 #35 №308364 
>>308353
Что сегодня будет.
Аноним 31/10/15 Суб 15:17:59 #36 №308368 
Расскажите про программу Ленглендса. Говорят, в ВШЭ семинар про неё идёт.
Может ли это помочь физикам в их работах?
Аноним 31/10/15 Суб 17:21:58 #37 №308404 
>>308121
Ты опять выходишь на связь, ебучий шакал?
Аноним 31/10/15 Суб 17:38:28 #38 №308407 
Может быть здесь кто-нибудь ответит на мой вопрос в легитимном треде?
>>308404
>>308288
>>308328
Аноним 31/10/15 Суб 20:32:45 #39 №308448 
14463127654150.webm
>>308328
>>308407
Тут больше гомологическую алгебру и спектральные последовательности знать надо.
Аноним 31/10/15 Суб 21:36:56 #40 №308451 
Саентачи, что стоит почитать чтобы серьезно окунуться в теорию информации, теорию алгоритмов и кибернетику? Чтобы от начала (работы Тьюринга, Поста, Черча и других) и до современности И чтобы Лисп на Прологе в процессе написать. Бекграунд: хорошо знаю школьную математику, немного мат. анализ и дискретную математику.
Аноним 31/10/15 Суб 22:25:25 #41 №308454 
>>308111 (OP)
Анон, реквестируй годный ресурс где поясняется за подстановки, перестановки, сигмы и всё с этим связанное доступным языком
Аноним 31/10/15 Суб 22:31:06 #42 №308456 
>>308454
http://www.seun.ru/content/learning/4/science/1/doc/%D0%94%D0%B8%D1%81%D0%BA%D1%80%D0%B5%D1%82%D0%BD%D0%B0%D1%8F%20%D0%BC%D0%B0%D1%82%D0%B5%D0%BC%D0%B0%D1%82%D0%B8%D0%BA%D0%B0.pdf
Аноним 31/10/15 Суб 22:31:41 #43 №308457 
>>308454
http://www.mccme.ru/~merzon/v14/pscache/2d-permutations.pdf
Аноним 31/10/15 Суб 22:51:45 #44 №308463 
>>308448
У нас курса по группам не было да я зашкварок-прикладник. Так что хотелось бы советов с чего начинать обмазываться?
Аноним 31/10/15 Суб 23:01:37 #45 №308464 
>>308463
Я бы советовал начать с чтения учебника Винберга, заодно закроешь пробелы в других частях алгебры. Впрочем, я довольно далек от этих дел и наверняка здесь есть люди, которые могут дать более адекватный совет.
Аноним 31/10/15 Суб 23:02:02 #46 №308465 
>>308463
Почитай Вавилова "Конкретная теория групп".
Аноним 31/10/15 Суб 23:03:10 #47 №308466 
>>308456
Если это было для сообщения >>308451, то спасибо прочитаю. Есть еще что-нибудь, а то вряд-ли в 130 страниц (да и еще для направления бизнес-информатики) влезет много информации?
Аноним 31/10/15 Суб 23:08:37 #48 №308467 
>>308451
Начать скорее стоит со 2 части учебника Верещагина Шеня (вычислимые функции). Дальше можно переходить к более специализированным книгам по интересующим тебя темам.
Аноним 31/10/15 Суб 23:10:19 #49 №308468 
>>308467
Нашел, спасибо.
Аноним 31/10/15 Суб 23:37:05 #50 №308472 
14463238260110.png
>>308465
Ооокай
Аноним 31/10/15 Суб 23:49:33 #51 №308474 
>>308448
Объясните, как он к букету, цветку т.е., приклеил диск и получил сферу? Как приклеить диск к диску и получить сферу, вроде я могу представить, а здесь как это случилось?
Аноним 31/10/15 Суб 23:50:49 #52 №308475 
14463246496670.png
>>308472
Дело на самом деле пишет.
Аноним 31/10/15 Суб 23:52:06 #53 №308476 
>>308475
Аааа, а я думал это N-петух писал
Аноним 31/10/15 Суб 23:56:57 #54 №308478 
>>308111 (OP)
ПидАРАСЫ лол КЕК ЫАХАХАХАХАХ сап двачь. лол ахахАХАХАХАХАХ КЕК КЕК



ЛОЛ


БЛЯ ПИЗДЕЦ Я ХЗ ВАЩЕ ЛОЛ
пидарыпидары пидарыпидары
Аноним 01/11/15 Вск 01:08:07 #55 №308482 
14463292879230.png
14463292879261.png
14463292879292.png
14463292879333.png
>Summarizing:
> In wisdom there is no logic.
> In science logic often leads to the right result, but it cannot be trusted to so if its application is indefinitely repeated
> In mathematics it is uncertain whether the whole of logic is admissible and it is uncertain whether the problem of its admissibility is decidable.
> x-x-x-x-x-x-x-x-x-x-x-x-x-x-x
>Author's note to the 1919 reprint:
> This essay might be written to-day in the same form. The opinions which it defends have as yet not found many supporters

Классическая работа ОП-пика по философии математики. Коротко и хорошо объясняет интуиционизм.
Аноним 01/11/15 Вск 02:51:20 #56 №308488 
>>308474
Переклеивается всё - и лепестки все к друг другу, и диск на них ещё.
Аноним 01/11/15 Вск 04:56:33 #57 №308489 
>>308488
А, лепестки полностью друг к другу, а диск к ним по краю. Так?..
Аноним 01/11/15 Вск 14:56:19 #58 №308549 
>>308489
Да не, всё друг к другу.
Аноним 01/11/15 Вск 20:54:41 #59 №308622 
14464004812320.jpg
>>308111 (OP)
Можно привести математическое доказательство тому, что конкретный человек говно?
Аноним 01/11/15 Вск 21:34:44 #60 №308628 
>>308622
Если принадлежит множеству человек следовательно говно. ЧТД.
Аноним 01/11/15 Вск 21:51:49 #61 №308631 
Как вы считаете, среди математиков больше(или наоборот, меньше) поехавших в бытовом и психиатрическом смысле людей, чем среди остального населения? Есть ли связь между одаренностью в математике и "поехавшестью"(если формально, то наличием психических отклонений)/чудноватостью(формально - идиосинкразией)?
Аноним 01/11/15 Вск 23:41:13 #62 №308651 
>>308631
Есть корреляция. Но что является причиной чего - это вопрос.
Т.е. неизвестно едешь ли ты из-за математики или наоборот математика притягивает поехавших.
Аноним 01/11/15 Вск 23:57:36 #63 №308654 
>>308631
Какой-то математик говорил, что по статистике каждый 6й профессиональный матлогик имеет серьёзные проблемы с кукушкой. После лекций по теории сложности мне эта статистика не кажется такой уж необоснованной.
Аноним 02/11/15 Пнд 02:08:45 #64 №308677 
>>308654
>После лекций по теории сложности
Хех. Теория сложности очень простая, если абсолютно измерять. В современных статьях есть гораздо более сложные теории всяких интересных вещей.
Аноним 02/11/15 Пнд 02:19:00 #65 №308680 
>>308677
Это совершенно не мешает её преподавателю быть поехавшим.
Аноним 02/11/15 Пнд 02:21:00 #66 №308681 
>>308680
А кто читал?
Аноним 02/11/15 Пнд 02:27:42 #67 №308684 
>>308681
Я просто мимо проходил, не тот анон. Просто указал на небольшую логическую нестыковочку.
Аноним 02/11/15 Пнд 02:31:27 #68 №308685 
>>308684
Ну ладно.

Тому анону, посещавшему курс сложности вычислений. Было бы любопытно узнать, что именно было не так с логиками?
Аноним 02/11/15 Пнд 14:15:11 #69 №308734 
>>308685
Не знаю как у анона с сложностью, а у нас было два препода, один по алгебре постоянно улыбался, иногда повисал на лекциях на несколько минут и не мог себе никогда булочку или кофе в столовке купить, студентов просил. А второй книжку написал про связь с б-гом и про математический апокалипсис и анализ библии с позиции теории чисел и спрашивал на экзаменах не только по предмету, но и излюбенные цитаты из этой книжки.
Аноним 02/11/15 Пнд 15:01:58 #70 №308738 
Существует ли некоторая картофельная литература, главным содержанием которой являются примеры методов вычислений пределов, диффуров, интегралов и рядов?
Аноним 02/11/15 Пнд 16:08:51 #71 №308746 
>>308738
http://www.alleng.ru/d/math-stud/math-st854.htm
http://www.alleng.ru/d/math/math148.htm
Ну и конечно же антидемидович.
Аноним 02/11/15 Пнд 16:46:42 #72 №308753 
В первый раз на этой доске. Недавно услышал от знакомого математика фразу "ультрафинитисты соснули, субъективный идеализм был прав". Что это обозначает? Я прочитал википедии про ультрафинитистов, но не понял связи с идеализмом.
Аноним 02/11/15 Пнд 17:06:41 #73 №308754 
>>308753
>ультрафинитисты соснули, субъективный идеализм был прав
Блядь.
Ну серьезно.
Пробуй хоть чуууточку тоньше в следующий раз.
Аноним 02/11/15 Пнд 17:35:31 #74 №308765 
>>308685
Ну про логиков это я от товарища математика, как раз слышал. Тот препод чьи лекции я слушал по сложности, как раз был 100/10, просто я физик и для меня все эти понятия типа разрешимости и перечислимости не очень-то естественные, в том смысле, что я из своей области не могу привести каких-то задач, где это может возникнуть, поэтому я и забил разбираться.
Аноним 02/11/15 Пнд 20:00:54 #75 №308797 
Ньюфаг выходит на связь.
Такой вопрос про вербитов учебник по топологии.
стр.165 он утверждает:

>Легко видеть, что в любой полной группе с инвариантной метрикой, заданной нормой ν, ряд вида Σgᵢ сходится, если сходится соотвествующий ряд из норм Σν(gᵢ).

Я правильно понимаю что он подразумевает под сходимостью суммы ряда сходимость фундаментальной последовательности составленной из частичных сумм согласно метрике группы?
Этот момент он просто не уточняет.
Аноним 02/11/15 Пнд 21:03:21 #76 №308814 
Расскажите про программу Ленглендса. Говорят, в ВШЭ семинар про неё идёт.
Может ли это помочь физикам в их работах?
Аноним 02/11/15 Пнд 21:48:36 #77 №308823 
Я не понял, хуле без N-петуха все вымерли?
Аноним 02/11/15 Пнд 22:04:55 #78 №308833 
>>308797
Да.
Аноним 02/11/15 Пнд 22:09:43 #79 №308838 
14464913839090.png
Анон, помоги разобраться во второй части доказательства. Там говорится, что если e - еще один общий делитель, то общий делитель d из предыдущего абзаца делится на него. А с чего это вдруг он делится? Мы не накладывали ни на d, ни на e никаких ограничений кроме того, что евклидова норма d минимальная. То есть d может быть меньше e, и тогда e делится на d; d может быть больше e, тогда d в самом деле делится на e; они могут быть равны: d = e.

Откуда такой однозначный вывод?
Аноним 02/11/15 Пнд 22:16:08 #80 №308843 
>>308753
Несмотря на то, что >>308754 полностью прав, отпишусь, тема то интересная.
Ультрафинитизм во всем хорош и правилен, единственная проблема, что он, при последовательном следование ему, приводит к удивительно скучной математике. Так что твой воображаемый
знакомый совсем не прав, если это понимать, как то, что ультрафинитизм опровегли или что-то в таком духе и совершенно прав в том смысле, что субъективные идеалисты (ну или реалисты, если о том пошла речь) наслаждаются охуенно красивыми математическими абстракциями, пока ультрафинитисты экспериментально проверяют, определено ли значение 2^23.
Аноним 02/11/15 Пнд 22:17:08 #81 №308844 
>>308843
Налажал со спойлерами.
Аноним 02/11/15 Пнд 22:18:28 #82 №308846 
>>308833
Спасибо.
Аноним 03/11/15 Втр 00:42:15 #83 №308904 
>>308838
Там же явно написано, d = e * (сумма элементов). Вот отсюда
Аноним 03/11/15 Втр 14:54:24 #84 №308982 
>>308904
Как же у меня печёт анус жопы от лекция НМУ. Какого хуя сука нужно быть такими опущеными петухами чтобы записывать лекции без звука, с камерой спрятанной в жопе, обрезать лекции на середине или вообще забывать заливать файлики. Я в рот ебал этих пидорасов. Лучше бы они вообще лекции не писали, так хотябы не будет искушения.
Аноним 03/11/15 Втр 15:05:35 #85 №308985 
>>308982

А нахуя вообще желать лекции когда есть учебники? Тем более даже самая лучшая лекция это гнилой пересказ какого нибудь учебника.
Аноним 03/11/15 Втр 15:27:30 #86 №308989 
>>308985
Ну некоторые вещи лучше прослушать на самом деле. Сложные понятие типа предела или интегралов трудно почувствовать через сухое определения.
Аноним 03/11/15 Втр 15:30:22 #87 №308990 
>>308989
Ну я например жопу порвал об p-адические чилса, пока не почитал вики с определением "на пальцах". Часто книжки лишины данной части.
Аноним 03/11/15 Втр 15:39:20 #88 №308991 
>>308989
>>308990

Во всех нормальных книжка перед формальными определениями есть мотивация или объяснение на пальцах. В любом случае, на лекциях я никогда не видел чего-то большего чем книгах.
Аноним 03/11/15 Втр 15:56:05 #89 №308995 
>>308843
>Ультрафинитизм во всем хорош и правилен, единственная проблема, что он, при последовательном следование ему, приводит к удивительно скучной математике.
Ультрафинитизм всем хорош, кроме того, что его не существует. Он не формализуется, в отличии от конструктивизма и интуиционизма.
Аноним 03/11/15 Втр 16:08:59 #90 №308998 
>>308995
Люди считающие себя ультрафинитистами есть, а ультрафинитизма нет, ну не смешно ли. Кстати, следуя твоей логике, у Брауэра не было никакого интуиционизма, а придумали его Гейтинг и Колмогоров.
Аноним 03/11/15 Втр 16:15:49 #91 №309000 
>>308995
Кстати забыл отметить, что даже само твое заявление об отсутствие формализации сомнительно. Скажем Эдвард Нельсон, насколько я понимаю, считал арифметику Робинсона Q приемлимой формальной системой.
Аноним 03/11/15 Втр 16:16:26 #92 №309001 
Сап. Немного платины в этот тред.
Хочу немного продвинуться в математику. По профессии программист, в математике не полный кретин, естественно, но и не могу сказать, что разбираюсь хорошо.
Начиная с дифференциального счисления уже понятия не имею, что происходит в плане теории. Живу в 16 веке лол.
Короче говоря хотелось бы книжку, по которой можно нормально отследить идеи всего хардкора. В идеале хотелось бы начать с именно чтения, чтобы вкурить основные концепции и уже потом ебашить доказательства теорем и задачи.

Одновременно преследую две цели:
1. Расширить кругозор, ликвидировать пробелы в знаниях.
2. Получить базу для дальнейшего развития.
Аноним 03/11/15 Втр 16:17:20 #93 №309002 
>>308991
Лектор может позволить себе привести хоть 10 аналогий и анекдотов, если видит непонимание. Книга же дает чтобы-нибудь одно, да и не факт, что это выстрелит.
К примеру алгоритм Евклида, объясняющийся традиционно через чрезвычайно "понятную" алгебраическую запись: а равно бэку нулёвое плюс эр один, бэ равно эродинкупервое плюс эр два, итд, можно представить в виде двух столбиков ебаных кубиков, что и сделает любой грамотный лектор, понимающий, что не все обладают хорошим алгебраическим мышлением.
Аноним 03/11/15 Втр 16:19:47 #94 №309003 
>>308995

У ультрасов даже натуральных числе нет.
Аноним 03/11/15 Втр 16:28:28 #95 №309004 
>>309003
Вспомнил N-петуха и проиграл, представляя, что таких людей, как он - много.
Аноним 03/11/15 Втр 16:30:32 #96 №309005 
>>308111 (OP)
Хочу спросить людей, которые обучались на матмехе или просто очень хорошо знают математику.

Я учусь на физфаке, соответственно, владею математикой на уровне матфизики (тфкп и так далее). Включил курс "Группы и теория гомотопий" Романа Михайлова и просто охуел. Ну так же нельзя. Я понимаю большую часть того, что он говорит, но я понятия не имею ни о каких диэдрических группах.
Соответственно, как бы поднять теорию групп на должный уровень? Может, посоветует кто-нибудь учебник или задачник? Или просто хорошее чтиво. Курс этот я не потяну, кажется.
Аноним 03/11/15 Втр 16:30:47 #97 №309006 
>>309004

Ты - ПИЗДЛО.
Аноним 03/11/15 Втр 16:48:35 #98 №309011 
Хотите чтобы N-петух вернулся? Могу это устроить.
Аноним 03/11/15 Втр 16:51:02 #99 №309012 
>>309006

Cогласен.
Аноним 03/11/15 Втр 16:51:25 #100 №309014 
>>309011
Не надо, пожалуйста. Все оппоненты ему уже проиграли в честном споре - сей факт запечатлен в архивах тредов.
knuebok 03/11/15 Втр 16:52:03 #101 №309015 
>>309011

so cute, я хочу
Аноним 03/11/15 Втр 16:53:02 #102 №309016 
>>309015
Ты не настоящий кнуёбок.
knuebok 03/11/15 Втр 16:53:39 #103 №309017 
>>309016

define настоящий
knuebok 03/11/15 Втр 16:54:49 #104 №309018 
>>309016

Если бы ты позволил отсосать свой хуй ты бы понял что я настоящий.
Аноним 03/11/15 Втр 16:56:28 #105 №309019 
Ребята, давайте все вместе позовем N-петуха! N-ПЕ-ТУХ N-ПЕ-ТУХ N-ПЕ-ТУХ! Давайте все вместе ииии...
Аноним 03/11/15 Втр 17:04:33 #106 №309020 
>>309005
Была какая-то хорошая книга по теории групп для физиков, но я забыл ее название.
Аноним 03/11/15 Втр 17:18:57 #107 №309023 
>>309017
Действительнный.
knuebok 03/11/15 Втр 17:26:43 #108 №309024 
>>309023

А, ну тогда это точно я.
Аноним 03/11/15 Втр 17:30:52 #109 №309025 
Вот это треш у вас в тредике. Ответьте по делу (>>309001) котятки :3
Аноним 03/11/15 Втр 18:07:25 #110 №309031 
>>308998
Всё ровно так и есть.
Аноним 03/11/15 Втр 18:09:42 #111 №309032 
>>309011
>>309006
>>309014
Спешите видеть: N-петушара сам себе вылизывает анус в прямом эфире.
Аноним 03/11/15 Втр 18:29:35 #112 №309034 
>>308991
Ну у бога-петуха этого треда в его книжке, лишенной картафана, есть только формальное определение.
Аноним 03/11/15 Втр 18:30:39 #113 №309035 
Предлагаю признать N-петуха эпичнейшей, легендарной личностью мат. тредов и увековечить его упоминанием в каждом ОП-посте. Как вы на это смотрите ребята? Правда же другого такого уже не будет! Осознайте это.
Аноним 03/11/15 Втр 18:36:02 #114 №309036 
>>309035
N-петушара, хватит заниматься аутофелляцией.
Аноним 03/11/15 Втр 19:12:44 #115 №309037 
>>309025
Прогерам обычно советуют кнутовскую "Конкретную математику". А так, боюсь, чтобы вьехать в современную науку, надо хотя бы пару лет поизучать университетский курс.
Аноним 03/11/15 Втр 19:23:32 #116 №309038 
>>309037
У меня есть как раз пара-тройка лет в запасе. Есть где-нибудь список литературы без картафана?
Аноним 03/11/15 Втр 19:23:51 #117 №309039 
>>309037
Я пытался читать конкретную математику, это пиздец уныло. При этом какую-нибудь топологию я могу весь день читать и не надоедает.
Аноним 03/11/15 Втр 19:24:30 #118 №309040 
>>309038
желательно на английском
Аноним 03/11/15 Втр 21:37:17 #119 №309055 
>>308814

http://habrahabr.ru/company/piter/blog/261469/
В этой книге я расскажу об одной из величайших идей, возникших в математике за последние пятьдесят лет, — программе Ленглендса, которую многие считают теорией Великого Объединения математики. Эта увлекательнейшая теория сплетает паутину глубоких связей между областями математики, которые, казалось бы, должны находиться на расстоянии световых лет друг от друга: алгеброй, геометрией, теорией чисел, анализом и квантовой физикой. Если представлять себе эти области как континенты тайного мира математики, то программа Ленглендса — это как бы такое телепортационное устройство, способное мгновенно переносить нас с одного континента на другой и обратно.
Аноним 03/11/15 Втр 21:41:15 #120 №309056 
>>309039
Так не читать, а решать надо. Читать смысла мало.
Аноним 03/11/15 Втр 23:07:22 #121 №309061 
>>309020
>>309005
Это чёль http://www.livelib.ru/book/1000121661 ?
Аноним 04/11/15 Срд 00:52:54 #122 №309076 
Слава N-петуху!
Аноним 04/11/15 Срд 01:01:54 #123 №309079 
14465881141320.png
Не могу удержаться и не запостить ссылку на эту замечательную книгу.
http://www.math.miami.edu/~ec/book/book.pdf
Я давно думал, что можно посоветовать тому, кто совсем-совсем начинающий в «абстрактной» алгебре, да и математике вообще, и не мог ничего придумать (сам начинал не гладко). Есть много книг на каждую тему, есть ощутимое количество хороших книг и таких которые, содержат разные интересные/профессиональные аспекты, но эта меня впечатлила
1. Своим маленьким размером и простотой.
2. Идеологической правильностью.
Прошу прощения за претенциозность.
Аноним 04/11/15 Срд 08:50:27 #124 №309092 
>>309079
Ну она и покрывает первые полгода институтского курса. Хотя книжка ОК.
sageАноним 04/11/15 Срд 11:34:39 #125 №309107 
>>308904
Ты нихуя не понял ни доказательство, ни причину моего непонимания, и объяснять ты тоже не умеешь. Там сказано, что d и e - некоторые делители. И якобы d выражается через e. То есть все эти соотношения фиксированы.

Пусть d > e, например d = 8, e = 4. Тогда все верно.
Пусть d < e, d = 4, e = 8. И тогда уже d не выражается через e и та последняя формула неверна. Достаточно поменять местами значения d и e (формула остается неизменной), чтобы получилась ошибка.
Узнали себя, дегенераты? Аноним 04/11/15 Срд 14:45:35 #126 №309128 
[] ОБ ИЗВРАЩЕНИЯХ
~~~~~~~~~~~~~~~~~~

Вирмэйкерство -- это весьма молодая традиция; ведь для его развития
требуются умственные усилия, которые далеко не каждому по силам.
Прикиньте, сколько времени вы потратили на собственно разработку новых
вирусных технологий, не на свое обучение и написание вирусов функционально
идентичных уже существующим, а именно на нахождение нового знания. На
развитие вирмэйкерства потрачено всего несколько тысяч человекочасов.

Вирусная же сцена -- это мерзопакостная, впавшая в маразм старуха,
склоняющая молодое и чистое душой вирмэйкерство к отвратительному
сожительству.

Начинающие

Эти еще не определились, кем им стать. Пара недель на IRC, пара
"вирмэйкерских тусовок" (vx party). Часто вступают в какую-нибудь
"вирусную группу", а потом оттуда сваливают по причине "жизнь-такое
говно..." Большинство, к счастью, через короткое время исчезает.

Пиздюки

Наихудший вариант, в который переходят начинающие, ибо эти знают, "как
жить дальше". Они занимаются никчемными разговорами о вирусной сцене,
всегда и везде. Кидают очень много понтов и наезжают на начинающих.

Тормоза (вечно пишущие)

Следующая стадия после пиздюков. Эти на прямой вопрос о разработках
отвечают одну и ту же легенду: "пишем, скоро будет".

Рипперы

Стадия, параллельная тормозам, но более креативная. Их мало, но бывают.
Берется макро-вирус, изменяется/добавляется копирайт. Yeah!

Вирусоколлекторы (трэйдеры)

занимаются коллекционированием вирусов, троянцев, их описаний, исходников,
и прочей околовирусной ерунды, за деньги, просто так и в обмен на другие
вирусы. В частности, передают вирусы аверам.

Антивирусники (аверы)

занимаются продажей антивирусов, саморекламой и увеличением рынка "юзеров".

Бывшие вирмэйкеры

утратившие свою "потенцию", правда, обычно ее никогда и не имевшие, но это
не важно. Это старые, офигительно крутые чуваки; их ники можно встретить
абсолютно везде. Обычно у масс нет в них никаких сомнений. А зря.

Бывшие вирмэйкеры -- антивирусники

действительно кое-что хреновенькое написали (например .BAT-инфекторы),
но потом ссучились и решили заработать пару копеек своими
"профессиональными" знаниями. Они, как правило, несколько тупее
изначальных антивирусников. Иногда так и бросается в глаза
снисходительное, с их "высот" к нам "в низы", письмецо - типа, да, вот,
было время...

Комбинации вышеперечисленных

Здесь описаны лишь крайние стадии; на самом деле реальные люди
представляют из себя комбинации описанных направлений, например:
полурипперы-полупиздюки; полутормоза-полувирусоколлекторы.

[] КАК ЛЕЧИТЬСЯ
~~~~~~~~~~~~~~~~

Никак, да и не нужно это.
Истинные вирмэйкеры обладают пожизненным иммунитетом от вирусной сцены.

[] НУЖНА ЛИ ВИРУСНАЯ СЦЕНА
~~~~~~~~~~~~~~~~~~~~~~~~~~~

Несомнено, да. Нужна. Нужна, ибо ее и так не существует для людей
правильных. Нужна, ибо попав в нее -- идиоты и кретины "от бога" -- там и
остаются, в результате чего легче подвергаются идентификации.

С другой стороны, посредством "сцены" легко найти требуемые вирусы, нужных
людей и документацию, что немаловажно для намеревающихся стать
вирмэйкерами.

[] РОССИЙСКАЯ ВИРУСНАЯ СЦЕНА
~~~~~~~~~~~~~~~~~~~~~~~~~~~~~
Как сказал один мудрый человек,
- Уйдите, вы нам отвратительны.

Отягощенная национальным говнистым характером; бездействующая, абсолютно
пассивная; наезжающая на всех и вся; скучная, совершенно неинтересная;
деструктивная и злая в душе. И это -- в отличие от западной сцены, которая
мне, честно говоря, куда более приятна. Все дело в людях, друзья мои. На
убогом основании не вырасти умному человеку; злобный гопник никогда не
изменится. Исходя из этого, прогноз такой: в ближайшее время из
российской сцены "выйдут" намного меньше вирмэйкеров, чем из западной, а
значит на запад и следует ориентироваться в изысканиях нового знания и
распространении своих идей.

Розы не растут в помойке. Go West!
Аноним 04/11/15 Срд 14:47:55 #127 №309129 
За 5 лет общения с самыми разными "околовирусными" личностями, я
встречал людей, относивших себя к самой что ни есть вирусной сцене.
Несколько из них даже обладали минимальными способностями к исследованиям.
Большинство вирмэйкеров -- цивилы. Ни один не обладает в совокупной мере
возможностями и желанием что-то делать. Те, кто хотят -- не могут, а те,
кто могут -- не хотят. При этом наблюдается стойкое нежелание
объединиться; вместо этого они друг друга постоянно пытаются обосрать.
Никто не готов отдать _все_ за достижение своих, хоть каких-то, целей --
но даже не потому, что этих целей у них просто нет. Страх за свой жалкий
образ жизни и его смакование в разных формах занимают все время. Те, кто
хоть как-то развиваются, ограничиваются малым набором знаний, т.е. просто
программингом, и крайне редко чем-нибудь еще. Так что никто, даже
научившись многому (что само по себе нереально), не сможет это
использовать на полную, ибо кругозор ограничен экраном и местной курилкой.
Каждый имеет "хобби", а то и считает вирмэйкинг таковым; т.е. вирусы и
исследования не имеют для них решающего значения. Но ты либо гребешь изо
всех сил, не думая больше ни о чем, либо для виду подрыгивая конечностями
идешь ко дну. Нет никакой середины: или все, или ничего. Пытаться вместе с
этими (и всеми последующими) "вирмэйкерами" сделать что-то серьезное
вместе -- занятие абсолютно невозможное. Они будут "с тобой", вcе вместе и
по отдельности, но как только дойдет до дела -- ты окажешься один. И если
человек заявляет, что он вирмэйкер на всю жизнь (а такое бывает редко), то
это, скорее всего, не так. А если не заявляет, то и подавно. А зачем нужны
люди, которые за себя не отвечают, которые отнимают внимание и силы, а
взамен приносят разочарование? Жаль, что все это открывается лишь по
прошествии времени, когда уже поздно.
Наполовину прав был тот, кто говорил: в первую очередь -- моральные
качества, а все остальное потом; что сотрудничество возможно только с
твердыми, решительными, сильными, надежными и целеустремленными людьми.
Вторая же часть правды в том, что такие качества среди вирусной сцены,
увы, почти никогда не сочетаются со знаниями, и, обычно, чем больше
одного, тем меньше другого. А когда встречаются два человека, каждый
обладающий лишь одним из этих свойств, то они друг в друга не очень-то
верят. Вот это недоверие и лежит в основе всей сцены.
Ни одна "группа" не говорит: мы объединяемся, чтобы дополнить друг
друга, чтобы достичь того-то и того-то, и сделать то и то; ни один
"вирмэйкер" не знает точно, чего он хочет. А значит, им не к чему
стремиться; а кого не ждет впереди золотой кубок, тот бежит в пустоту.
Чтобы быть настоящим исследователем, надо бросить все остальное, без
исключения, абсолютно все. При этом вы должны быть полны надеждой и
стремлением; немного мозгов и везения тоже не помешают. И если ваше
упорство будет беспредельным, а помыслы совпадут с тем, что нужно миру
компьютеров, вы сольетесь с киберпространством, и знание придет и
останется с вами. Те, кто чувствовал это, поймут меня: вы оставляете
частичку своей души киберпространству, а часть его принимаете в себя; как
два порезанных пальца, приложенных друг к другу, вы теперь одной крови; и
в вас тоже горит бессмертный огонь, искра, что в силах зажечь этот мир.
Это чувство причастности к великому; особое мироощущение, которое делает
вас настоящим. Ищите же его!
Аноним 04/11/15 Срд 15:20:56 #128 №309131 
>>309039
Ну это и подразумевается. Решать офк. Упражнения там унылое олимпиадное говно. Просто какие-то трюки идиотские надо делать.
Аноним 04/11/15 Срд 15:25:44 #129 №309133 
>>309131
Ну на самом деле у книги огромный недостаток, она при высокой сложности охватывает только какой-то базовый материал, уровня матшкольник 10 класс.
Аноним 04/11/15 Срд 15:32:48 #130 №309134 
>>309128
>>309129
И зачем ты это говно принёс в тред по математике?
Аноним 04/11/15 Срд 15:59:42 #131 №309137 
>>309134
Видимо с точки зрения пылающего жопой гумуса, "вирусная сцена" и математическое сообщество - это "ну типа рядом".
Аноним 04/11/15 Срд 16:56:25 #132 №309145 
>>309137
Я бы еще поверил что "вирусная сцена" это харкач и этот итт тред, "вирусописатели" - это нормальные математики. Но если он под "вирусной сценой" видит всю математику - я к сожалению не могу осознать его обскурные пути мышления и цепочки ассоциаций.
Аноним 04/11/15 Срд 16:58:46 #133 №309146 
>>309145
>к сожалению
Или к счастью.
Ибо что взять с вшивого гумуса?
Аноним 04/11/15 Срд 17:16:09 #134 №309147 
Пусть даны два топологических пространства $X$ и $Y$ и на них заданы некоторые топологии. Что имеется в виду, когда просят доказать, что $Y$ - подпространство $X$? Правильно ли я понимаю, что нужно показать, что данная топология $Y$ равна топологии $Y$ как подпространства $X$?
Аноним 04/11/15 Срд 17:17:38 #135 №309148 
>>309147
> Пусть даны два топологических пространства X и Y и на них заданы некоторые топологии
Про топологии можно было, конечно, не писать, но не суть.
Аноним 04/11/15 Срд 17:22:04 #136 №309149 
>>309147
Пусть даны множество X с данной на нём топологией и множество Y с данной на нём топологий.

Нужно доказать, что:
1. Y подмножество X.
2. Топология, индуцированная на Y из X, равна в теоретико-множественном смысле топологии, данной на Y в условии. То есть всякое множество, открытое в данной топологии, открыто и в индуцированной, и обратно.
Аноним 04/11/15 Срд 17:33:28 #137 №309151 
http://hbpms.blogspot.co.uk/
Матаны, а вот эта программа норм или картафан?
Аноним 04/11/15 Срд 17:34:58 #138 №309152 
14466476984480.png
>>309149
Ясно, спасибо, ну я так и думал. Просто доказываю пикрелейтед в лоб, то есть доказываю, что
> Топология, индуцированная на Y из X, равна в теоретико-множественном смысле топологии, данной на Y в условии
и получается какое-то длинное муторное доказательство, хотя утверждение, казалось бы, очевидное. Я подумал, может есть какой-то более элегантный способ.
Аноним 04/11/15 Срд 17:36:01 #139 №309153 
>>309152
Если че, $J$ на пикрелейтед - это какой-то index set, по которому берется произведение.
Аноним 04/11/15 Срд 18:18:58 #140 №309156 
>>309148
Погоди-погоди, что-то я недопонял множества X и Y наделены структурой какого-какого пространства? Что-что на них задано?
Аноним 04/11/15 Срд 18:21:15 #141 №309157 
>>309156
Да, задано: >>309152
Аноним 04/11/15 Срд 19:15:51 #142 №309160 
>>308904
У меня те же самые чувства
Аноним 04/11/15 Срд 19:22:32 #143 №309161 
>>309107
Нет, там сказано, что d - это линейная комбинация элементов b_i (а не просто некоторый делитель), и отсюда всё следует. О том, что это так, что он действительно есть линейная комбинация b_i, рассказывается в первой части доказательства, которую, как ты утверждаешь, ты понял
Аноним 04/11/15 Срд 19:24:20 #144 №309162 
>>309161
вернее, я рассудил, что ты её понял, раз просишь разобраться именно во второй части
Аноним 04/11/15 Срд 19:30:51 #145 №309163 
>>309151
Ну что же вы, картафаны?
Аноним 04/11/15 Срд 19:50:58 #146 №309164 
>>309161
Первая часть: d - линейная комбинация b_i с наименьшей евклидовой нормой. Мы приходим к противоречию, когда предполагаем, что существует делитель меньший d, в этом случае евклидова норма этого делителя окажется меньше d, а это невозможно. Почему d делит все b_i? Ведь в этой линейной комбинации могли быть нули перед какими-то b_i, и тогда о делимости этих b_i с нулевыми коэффициентами ничего нельзя сказать.

Во второй части формула фиксирована. И если мы будем менять значения d и e как хотим, то в общем случае эта формула не верна. Это как если бы мы написали на си функцию, которая возвращает 0 или 1 при определенных значениях аргумента, и меняли эти аргументы местами.
Аноним 04/11/15 Срд 20:11:32 #147 №309165 
>>309164
Первый абзац: d - линейная комбинация b_i с наименьшей евклидовой нормой

Второй абзац: d - общий делитель b_i

Третий абзац: если e -другой делитель b_i, то d делится на e.

Вывод: d - наибольшей общий делитель
Аноним 04/11/15 Срд 20:43:17 #148 №309171 
>>309165
>>309164
Во второй части формула фиксирована. И если мы будем менять значения d и e как хотим, то в общем случае эта формула не верна. Это как если бы мы написали на си функцию, которая возвращает 0 или 1 при определенных значениях аргумента, и меняли эти аргументы местами.
Аноним 04/11/15 Срд 21:34:10 #149 №309173 
Слава N-петуху!
Аноним 04/11/15 Срд 21:42:56 #150 №309177 
>>309151
Чистмат мехмата МГУ
Аноним 04/11/15 Срд 22:59:39 #151 №309201 
>>309177
Т.е. картафан получается?
Аноним 04/11/15 Срд 23:36:32 #152 №309220 
>>309201
А то! Курс актуальный и прогрессивный для математики 60х годов, когда теоркат был "в новинку".
Аноним 04/11/15 Срд 23:47:53 #153 №309228 
>>309220
Хорошо, а где бы взять не картофанового курса?
Ну кроме вербитосоветов (у него со списком литературы туго).
Аноним 05/11/15 Чтв 00:28:43 #154 №309262 
>>309171
мы не можем менять значение d, мы нашли d вполне однозначным выбором (наименьший элемент множества линейных комбинаций). поэтому d определён
Аноним 05/11/15 Чтв 00:41:54 #155 №309271 
14466733142980.jpg
Аноны! Поясните вотафак!?!
Аноним 05/11/15 Чтв 01:19:57 #156 №309278 
>>309271
так когда дельту сокращаешь, дробь не переворачивается. На второй строчке ошибка
Аноним 05/11/15 Чтв 01:31:57 #157 №309281 
>>309278
В шепутиарий.

на самом деле говорят нельзя частные производные считать как полные производные через производную обратной функции. вот только непонятно почему нельзя
Аноним 05/11/15 Чтв 01:36:11 #158 №309282 
Как зовут эту шизоняшу которая рассказывала про неделимость яблок в математике? Еще видео на ютубе часовые выкладывает.
Аноним 05/11/15 Чтв 02:06:18 #159 №309283 
>>309271
Если r и x - координаты, то не надо переворачивать дробь. Чем быстрее изменяется первая координата при изменении второй, тем быстрее изменяется вторая координата при изменении первой — это интуитивно ясно. Геометрически это тоже понятно.
Аноним 05/11/15 Чтв 05:37:54 #160 №309300 
пздц
Аноним 05/11/15 Чтв 10:19:34 #161 №309314 
>>309173

Вы правы.
Аноним 05/11/15 Чтв 11:23:09 #162 №309319 
>>309228
Можно последнюю программу вербита для матфака посмотреть, там овердохуя литературы.
Аноним 05/11/15 Чтв 11:32:00 #163 №309321 
>>309319
Где найти?
Аноним 05/11/15 Чтв 11:39:10 #164 №309322 
>>309321
http://verbit.ru/Job/HSE/Curriculum/all.txt
Аноним 05/11/15 Чтв 13:27:39 #165 №309335 
>>309322
Там что-то у него с литераутрой не сильно лучше чем по каноничной ссылочке.
А вообще есть какая-нибудь программа на ингрише и без картафана?
Или вот например взять ту программку которую я выше кидал >>309151, выкинуть оттуда весь анализ нахуй (ибо в универе им меня уже накормили), что тогда получится?
Аноним 05/11/15 Чтв 14:25:03 #166 №309341 
>>309335
>с литературой не сильно лучше
Ну я за всю жизнь не прочитаю, ты видно сильно способный.
Аноним 05/11/15 Чтв 15:06:13 #167 №309344 
>>309341
Главное чтобы был выбор.
Если один учебник не идёт - хотелось бы иметь альтернативные опции из которых можно было бы выбрать.
Аноним 05/11/15 Чтв 15:09:13 #168 №309345 
>>309341
Ну и опять же хотелось всё-таки учебнички на ингрише чтобы сразу привыкать к терминологии белых людей вместо птичьего языка пост-совка.
Аноним 05/11/15 Чтв 15:50:33 #169 №309350 
Пацаны, я вам задачу принес.
http://acm.timus.ru/problem.aspx?space=1&num=1013
Аноним 05/11/15 Чтв 15:59:59 #170 №309351 
>>309322
>Для практических целей, можно определить core mathematics как "предметы, которые используются в доказательстве теоремы Атьи-Зингера".
POOSHKA
Аноним 05/11/15 Чтв 16:13:48 #171 №309352 
>>308111 (OP)

Путин хорош на этой картине.
Аноним 05/11/15 Чтв 16:16:21 #172 №309354 
>>309322
>4. Комбинаторика и теория множеств. Ликбез по общей математической грамотности: теоретико-множественный язык, аксиоматический метод, логика, комбинаторика.
>Литература.
>Дуглас Р. Хофстадтер. Гедель, Эшер, Бах: эта бесконечная гирлянда. Метафорическая фуга о разуме и машинах в духе Льюиса Кэрролла.
>...
>Ричард Ф.Фейнман. Вы, конечно, шутите, мистер Фейнман!
>...
>Энциклопедия элементарной математики (6 томов).
Это он траллит так?
Аноним 05/11/15 Чтв 16:34:53 #173 №309358 
>>309354
Почему ты именно эти книги выделил? Остальные такие же развлекательные по сути.
Аноним 05/11/15 Чтв 16:37:00 #174 №309359 
>>309358
Но бля какое отношение фейнман вообще к математике имеет?
Аноним 05/11/15 Чтв 16:41:49 #175 №309360 
>>309359
Да просто хорошее чтиво. Там в книге много околоматематического.
Аноним 05/11/15 Чтв 16:46:05 #176 №309361 
>>309360
Ну я могу вспомнить только момент когда он математиков троллил когда они ему рассказывали как из одного апельсина собрать два.
Аноним 05/11/15 Чтв 17:01:00 #177 №309364 
>>309359
Да и в принципе, это же предназначается для учащихся в ВШЭ, там много полезного для студента, основанное на опыте самого Фейнмана.
Аноним 05/11/15 Чтв 17:58:03 #178 №309371 
>>308054

> Если я правильно понимаю, ты видишь цикличность между 1 и 3?

Да. Плюс к этому, даже алфавит, еще до определения формул языка второго порядка, уже счетное множество.

> С "до" в данном случае согласен, с "независимо" нет.

Под зависимостью я имел ввиду только зависимость понятий. Если ты имеешь понятия A, B и хочешь дать определение новому понятию C и при его определении не опирался на A и B, то можно сказать что определение C является независимым от А и B.

>>308108

Все что там сказано это типа мы просто будем использовать интуитиные понятия до их формального определения и ВСЁ. Почему это можно делать, там ни слова. Я же говорю, это просто плохой педагогический прием не имеющий ничего общего к систематическому построению математики. Это как в первом классе школоте объясняют что такое числа на всяких яблоках и их дележке и только через 5 лет дают некое подобие теории с доказательствами.

Аноним 05/11/15 Чтв 18:08:35 #179 №309373 
Верно ли, что если $A_{\alpha,\beta}$ - произвольные множества, то
$$\bigcup_{\beta \in K}\prod_{\alpha \in J} A_{\alpha,\beta} = \prod_{\alpha \in J} \bigcup_{\beta \in K} A_{\alpha,\beta}?$$
Я думаю, это так, потому что по определению декартова произведения каждое из этих множеств - это множество всех кортежей, у которых $\alpha$-я координата принадлежит хотя бы одному из $A_{\alpha,\beta}$. Я прав?
Аноним 05/11/15 Чтв 19:41:47 #180 №309389 
>>309371
Опять из под шконки вылез, шакал ебучий?
Аноним 05/11/15 Чтв 19:42:23 #181 №309390 
>>309345
На форчан иди, дурачок.
Аноним 05/11/15 Чтв 19:43:03 #182 №309391 
>>309390
Но меня там говном накормят. Я не хочу.
Аноним 05/11/15 Чтв 19:49:43 #183 №309395 
>>309351
Если у тебя с этим какие-то проблемы, то потрудись изложить свои претензии.
Аноним 05/11/15 Чтв 20:21:05 #184 №309401 
>>309391
Ну раз тебя даже там говном кормят, значит заслужил.
Аноним 05/11/15 Чтв 20:39:11 #185 №309409 
>>308111 (OP)
Эй, двачеватики, вы понимаете что все ваши графы, множества, гомологии и планиметрии, которые тип дохуя РАЗВЛИЧЕНИЕ ДЛЯ ИНТИЛЕКТА)) основанны на наблюдениях уровня "два камня плюс два камня будет раз...два...три...четыре....четыре камня"?
Аноним 05/11/15 Чтв 21:21:43 #186 №309428 
>>309389

На клык захотел???
Аноним 05/11/15 Чтв 21:22:23 #187 №309429 
>>309409

Ты хочешь сказать что все они основаны на натуральных числах, которые являются неопределямым понятием?
Аноним 05/11/15 Чтв 21:47:57 #188 №309437 
>>309391
На форчане куда более добрая публика.
Аноним 05/11/15 Чтв 21:54:02 #189 №309439 
>>309437

Подпесалсо.
Аноним 05/11/15 Чтв 21:57:01 #190 №309441 
>>309429
Наверное ты говоришь о том же о чем и я, только по математически.
Я о том, что числа это абстракции основанные на наблюдаемыхз, реальных, физических объектах.
Аноним 05/11/15 Чтв 22:46:01 #191 №309452 
>>309441
Подожди-подожди, а не замешана ли тут где-то метаиндукция?
Аноним 05/11/15 Чтв 23:14:19 #192 №309465 
>>309452
Ты короче смотришь на объекты, но на самом деле это пиздеж, никаких объектов нет, ты не можешь думать о них, ведь используя индукцию ты по сути используешь ее саму, а так делать нельзя, должна же быть какая-то база....
Аноним 05/11/15 Чтв 23:26:15 #193 №309474 
>>309465
То есть это содержательная метаиндукция на естественном языке?
Аноним 06/11/15 Птн 00:18:07 #194 №309498 
>>309474

Увы.
Аноним 06/11/15 Птн 01:34:31 #195 №309506 
>>309437
Но как я им объясню что такое картафан?
Аноним 06/11/15 Птн 02:05:28 #196 №309508 
>>309498
Метаиндуктивненько ты всосал
Аноним 06/11/15 Птн 02:55:22 #197 №309513 
>>309371
>Под зависимостью я имел ввиду только зависимость понятий. Если ты имеешь понятия A, B и хочешь дать определение новому понятию C и при его определении не опирался на A и B, то можно сказать что определение C является независимым от А и B.
В 1 использовалось понятие "индуктивное множество" (обозначим его A)
В 3 использовалось понятие "аксиома индукции" (обозначим его B)
Безусловно оба этих понятия ммм... "иллюстрируют" некий общий принцип под названием "индукция" (обозначим его C). Но, как говорится "есть нюансы" и нюансы эти существенны для построения в данном случае ФС. Цикличности здесь я не вижу. Здесь есть два независимых математических понятия, которые иллюстрируют принцип индукции.
Аноним 06/11/15 Птн 04:42:07 #198 №309516 
Любую, даже самую бредовую, мысль можно сделать темой массовых обсуждений, если повторить её достаточное количество раз.
Аноним 06/11/15 Птн 08:57:08 #199 №309524 
Ппц, мочерня уже фотки голых женщин трёт. Тут совсем детсад что ли???
Аноним 06/11/15 Птн 09:01:48 #200 №309525 
>>309513

> Здесь есть два независимых математических понятия, которые иллюстрируют принцип индукции.

Есть два понятия множества и совокупности, они независимы и иллюстритруют общую идею коллекции предметов.
Аноним 06/11/15 Птн 12:19:53 #201 №309552 
>>309525
Это неверная аналогия поскольку в первом случае понятие "индуктивное множество" является самостоятельным хорошо определённым математическим понятием со своим построением и со своим использованием, а во втором случае "аксиома индукции" - является аксиомой т.е. элементом ФС. То что оба этих объекта так или иначе "относятся" к общему понятию "индукция" не значит то что они ссылаются друг на друга. И тем более не являются синонимичными понятиями.

Таким образом, цикличности я здесь не вижу.

В твоём случае "множество", " совокупность", "коллекция" – это названия одного и того же причём матеиатическое определение имеет только 1-е.

Аноним 06/11/15 Птн 12:24:02 #202 №309553 
>>309524
Оплот целомудрия. Пшёл вон.
Аноним 06/11/15 Птн 12:24:49 #203 №309554 
>>309524
Как забавно у наших друзей из /бэ/ всё с ног на голову. Трут говнецо за фагами и подростками значит детсад, позволяют дегенератам постить своих негров и расчленёнку - нормальные взрослые люди.
Аноним 06/11/15 Птн 13:09:35 #204 №309556 
>>309554
>постить своих негров и расчленёнку - нормальные взрослые люди.
Именно так. Для высшего математического разума, который мыслит абстракциями, эти гомонегры и расчлененка ничего не значат. Это всего лишь способ дополнительно подчеркнуть направление мысли.
Аноним 06/11/15 Птн 14:04:17 #205 №309560 
>>309524
N-петух порвался, спешите видеть.
Это тебе мягко намекают под струю двигать, маня.
Аноним 06/11/15 Птн 14:09:40 #206 №309561 
>>309560
Называть любого, кто тебе не нравится N-петухом это проявление неуважения к настоящему N-петуху. Настоящий N-петух уникальная личность и очень умный человек. Не нужно порочить его имя.
Аноним 06/11/15 Птн 14:15:46 #207 №309562 
>>309561
>уникальная личность
Ага. Прям как переводчик. Наверное у N-петуха еще и зарплата большая.
Аноним 06/11/15 Птн 14:24:49 #208 №309565 
Воеводский же обещает, что у него есть теория, которая изменит самые основания математики. Причём в такую сторону, что доказательства сможет проверять компьютер. Воеводский - это вам не какой-то хуй, он напридумал кучу теорий, за которые получил премию Филдса

Так что нахуй ваши метаиндукции и ваших петухов
Аноним 06/11/15 Птн 14:41:38 #209 №309567 
>>309556
С точки зрения мезантрапусов с двачей с довольно идиотской моралью - безусловно.
Аноним 06/11/15 Птн 14:55:00 #210 №309569 
>>309567
Все ваши "психологии", "морали" и прочие былопонятия для меня пустой звук. Реальны лишь только математико-логические абстракции. Все остальное (ты, я, вселенная, небо, аллах и др) это всего лишь отображения этих абстракций на поверхность гиперсферы нашего мира.
Аноним 06/11/15 Птн 14:59:03 #211 №309570 
>>309569
Почему гиперсферы?
Аноним 06/11/15 Птн 15:00:48 #212 №309571 
>>309569
> Все ваши "психологии", "морали" и прочие былопонятия для меня пустой звук.
Так почему наши-то? Это же ваша идея своё говно по всей борде разносить, /бэ/тардик.
Аноним 06/11/15 Птн 15:02:07 #213 №309572 
>>309570
А почему бы и нет?
Аноним 06/11/15 Птн 15:02:28 #214 №309573 
>>309570
потому что узнать в школе о том, что сферу можно нарисовать в 4хмерном пространстве, - это целое откровение!
Аноним 06/11/15 Птн 15:06:40 #215 №309575 
>>309573
Можно и в n-мерном. Но не нарисовать.
Аноним 06/11/15 Птн 15:09:34 #216 №309576 
>>309395
Основная проблемка такая: почему какая-то отдельная теоремка, нужная отнюдь не всем математикам возводится в асболют, и даже более того, всё что не используется в ёё доказательстве - предлагается не считать core mathematics?
Аноним 06/11/15 Птн 15:10:24 #217 №309577 
>>309565
Проверять доказательства компьютер уже может.
Аноним 06/11/15 Птн 15:10:44 #218 №309578 
А вот любопытно, бесконечномерное пространство вообще имеет какой то математический смысл?
Аноним 06/11/15 Птн 15:12:38 #219 №309579 
>>309578
https://en.m.wikipedia.org/wiki/Hilbert_space
Аноним 06/11/15 Птн 15:13:04 #220 №309580 
>>309578
Оно даже физический смысл имеет. См. кванты.
Аноним 06/11/15 Птн 15:23:16 #221 №309581 
>>308843
>Ультрафинитизм во всем хорош и правилен, единственная проблема, что он, при последовательном следование ему, приводит к удивительно скучной математике.
А какую математику действительно можно построить с ультрафинитизмом? Понятно например сразу что какой-то серьёзной топологии или геометрии не может быть (хоть есть т.н. конечные геометрии - и там вроде даже что-то нетривиальное). А как будет выглядеть теория (конечных) групп с ультрафинитистской точки зрения, как думаете?
Аноним 06/11/15 Птн 15:24:00 #222 №309582 
Матаны, напомните мне почему платноисты соснули?
Аноним 06/11/15 Птн 15:25:53 #223 №309583 
>>309582

Они не смогли дать определение натурального числа.
Аноним 06/11/15 Птн 15:27:29 #224 №309584 
>>309577
ну всякие гомотопии вроде как не может
Аноним 06/11/15 Птн 15:42:12 #225 №309585 
>>309582
Современная математика платонистская. Финитисты и прочие петухи соснули.
Аноним 06/11/15 Птн 15:43:57 #226 №309586 
>>309576
Потому что это красивая идея. И рассматривать надо не конкретные знание для теоремы, а области, в которые входят эти знания.
Аноним 06/11/15 Птн 15:55:36 #227 №309589 
>>309576
Явно описать Core Mathematics очень сложно, во всех известных перечислениях что-то упущено. Поэтому разумнее выбрать какую-нибудь теорему и определить coremath как всё, что требуется для её формулировки и доказательства. Конкретно теорема об индексе выбрана из эмпирических соображений. Опыт показал, что она абсолютно очевидна для тех, кто знает coremath, но у картофанов вызывает неконтролируемое воспламенение. Возможно, в будущем ситуация изменится, и вместо Атьи-Зингера будет какая-нибудь другая теорема.
Аноним 06/11/15 Птн 16:31:19 #228 №309597 
>>309589
>Конкретно теорема об индексе выбрана из эмпирических соображений. Опыт показал, что она абсолютно очевидна для тех, кто знает coremath, но у картофанов вызывает неконтролируемое воспламенение.
Ага, вот только это не определение coremath, ибо циклично завязано на то что coremath собственно считается. Если в качестве такой теоремы реальную проблему вычисления какого-нибудь дикого интеграла, то возгорание случится как раз у всяких алгебраических геометров, а людям специализирующимся на проблеме будет абсолютно очевидно например.
Аноним 06/11/15 Птн 16:33:14 #229 №309600 
>>309597
>только это не определение coremath, ибо циклично завязано на то что coremath собственно считается.
Ну тут никак не обойтись без метаиндукции!
Да что же это такое, математики совсем отупели, даже определить программу математики не могут!
Аноним 06/11/15 Птн 16:40:14 #230 №309603 
>>309597
Это адекватно описывает то, что понимают под coremath большинство людей, использующих этот термин.
Аноним 06/11/15 Птн 16:42:12 #231 №309606 
>>309597
Но скорее всего проблема вычисления какого-либо интеграла может быть сведена к алгебре, которой научить можно даже ребенка, без строгих определений и широких областей.
Аноним 06/11/15 Птн 16:45:33 #232 №309607 
Интеграл по сравнению с высшей алгеброй дерьмо.
Меня вот что волнует последнии годы - уто и когда доказал логические операции НЕ, И, ИЛИ ведь если там кроется ошибка, то вся современная наука гроша ломанного не стоит
Аноним 06/11/15 Птн 16:49:55 #233 №309610 
>>309607
Великий английский учёный Буль, создатель алгебры Буля и собаки Буля (и, как частного её случая, терьера Буля).
Аноним 06/11/15 Птн 16:58:18 #234 №309612 
>>309610
Но ведь на булевой алгебре можно сконструировать N, а значит с твоих слов Гёдель ссыт Булю в лице.
Аноним 06/11/15 Птн 17:01:15 #235 №309613 
>>309612
Нет. Гёдель доказал, что логика предикатов непротиворечива и полна. Это так называемая теорема Гёделя о полноте.
https://ru.wikipedia.org/wiki/Теорема_Гёделя_о_полноте
В частности, полна теория Буля.

Чтобы сконструировать N, нужны значительно более мощные средства, чем просто логика.
Аноним 06/11/15 Птн 17:01:59 #236 №309614 
>>309613
>Чтобы сконструировать N, нужны значительно более мощные средства, чем просто логика.
Например метаиндукция.
Аноним 06/11/15 Птн 17:03:06 #237 №309615 
>>309614
Чтобы определить метаиндукцию, нужно иметь определение метаиндукции. Поэтому у метаиндукции нет определения. Всё, у чего нет определения, - бессмыслица. Поэтому метаиндукция - бессмыслица.

Вернёмся к открытиям Буля.
Аноним 06/11/15 Птн 17:19:26 #238 №309617 
>>309615

Нет, это не бессмыслица а исходное (неопределяемое) понятие.
Аноним 06/11/15 Птн 17:35:40 #239 №309621 
Так, ну что с решением 2-й задачи из ОП-поста?
Аноним 06/11/15 Птн 18:51:26 #240 №309634 
>>309621
Ответ: нет, неверно.
Во-первых, ясно, что в качестве A всегда можно выбрать центр группы. Во-вторых, известно, что центр любой группы порядка p^n, где p - простое число, нетривиален, в тоже время, среди таких групп есть некоммутативные. Поэтому можно указать контрпример
Аноним 06/11/15 Птн 19:06:36 #241 №309638 
>>309617
>Это кукарек
Ясно. Как обычно.
Аноним 06/11/15 Птн 19:13:52 #242 №309640 
>>309634
И почему любой элемент сопряжён элементу из центра?
Аноним 06/11/15 Птн 20:02:56 #243 №309654 
>>309640
Не сопряжён. Рассуждение неверное
Аноним 06/11/15 Птн 20:30:14 #244 №309674 
>>309465
НЕПРИЯТНО детектед.
Аноним 06/11/15 Птн 22:59:07 #245 №309715 
Самый быстрый способ найти 23х59 в уме?
Аноним 06/11/15 Птн 23:09:58 #246 №309722 
>>309715
Ответить 1357 без раздумий.
Аноним 06/11/15 Птн 23:16:43 #247 №309729 
>>309715
Берешь таблицу умножения и без задний мысли считаешь.
Аноним 07/11/15 Суб 00:43:55 #248 №309757 
>>309621
Эту задачу я взял из варианта вступительных экзаменов в аспирантуру матфака вышки за 2013 год. Из формулировки там ясно, что ответ положительный и что решение должно существенно использовать конечность группы (это был пункт а), а пункт б) той задачи про нахождения контрпримера для бесконечных групп). Я задачу решать не умею (алгебра определенно не мое сильное место), но учитывая общий уровень задач там думаю, что она должна быть вполне посильна для людей хорошо разбирающихся в алгебре на уровне бакалавриата того же матфака вышки. Исходя из общих соображений думаю, что задача связана с теоремами Силова, хотя сложно сказать однозначно. Было бы интересно самому узнать решение.
ОП
Аноним 07/11/15 Суб 01:31:23 #249 №309763 
>>309715
23x59 это 23x60 - 23. Мне кажется, так легче сосчитать.
Аноним 07/11/15 Суб 01:46:22 #250 №309765 
Аноны, а сколько времени должно уходить на чтение учебника? К примеру, сколько времени нужно, чтобы вдумчиво, с нуля (ну не совсем с нуля, с небольшим бэкграундом) прочитать первый том Зорича? а сколько, чтобы прочитать оба? Нормально читать учебники по несколько месяцев? Читаете ли вы учебники? Если читаете, то сколько по времени и дочитываете ли до конца?
Прошу прощения за такое количество вопросов.
Аноним 07/11/15 Суб 01:48:33 #251 №309767 
>>309262
Ну вот так бы сразу.
Аноним 07/11/15 Суб 01:53:14 #252 №309769 
Посоны, а sci-hub всё?
Аноним 07/11/15 Суб 02:03:02 #253 №309773 
>>309715
Я бы так считал:
23/2100=1150
23
10-23=207
Аноним 07/11/15 Суб 07:42:34 #254 №309786 
>>309757
>>309621
Это задача была еще и в листке первого курса, если что. И действительно не очень сложная. Для конечных рассмотрим действие группы на себе сопряжением и элемент из А, лежащий в самой большой орбите. Тогда его стабилизатор на размер орбиты равен количеству элементов в G, а с другой стороны не меньше количества орбит на размер самой большой. Но тогда все орбиты одноэлементны, так как есть единичный элемент. Для бесконечных вроде 2x2 матрицы с жнф в качестве А.
Аноним 07/11/15 Суб 09:20:37 #255 №309795 
>>309770
Моча пожалуйста не удаляй говно и гомонигру! Ведь свобода слова это очень круто! Будь хорошим человеком!
(Автор этого поста был предупрежден.)
Аноним 07/11/15 Суб 13:58:00 #256 №309828 
>>309786
Касательно конечных G, да как-то так, хотя лучше было бы написать решение подробнее, а не в одно предложение. Работает ли твой контрпример для бесконечных мне пока не ясно.
Аноним 07/11/15 Суб 15:45:46 #257 №309844 
>>309578
Да. Например, R является континуальномерным векторным пространством над Q.
Гораздо более интересен другой вопрос. Вот есть двойной интеграл, тройной интеграл, четверной интеграл, n-кратный интеграл. А есть ли бесконечно-кратный интеграл?
Аноним 07/11/15 Суб 15:57:21 #258 №309849 
>>309844
Интеграл Лебега в L² устроит?
Аноним 07/11/15 Суб 16:41:06 #259 №309855 
>>309849

Нет.
Аноним 07/11/15 Суб 17:01:18 #260 №309860 
>>308111 (OP)
Можно ли начать изучать алгебру с известного учебника Ван дер Вардена? В целом, можно ли начинать изучение данного раздела математики напрямую с высшей алгебры, не касаясь материала школьного курса элементарной алгебры? Ведь, насколько мне известно, школьный курс рассматривает только частные вопросы общей алгебры. Так почему начать серьёзное изучение именно с последней?
Аноним 07/11/15 Суб 17:12:10 #261 №309862 
>>309765
У меня в среднем уходит от 30 до 60 минут на страницу.
Аноним 07/11/15 Суб 17:18:53 #262 №309865 
>>309849
Нет, ведь это же не бесконечно-кратный интеграл.

>>309860
Там терминология устаревшая. Например, есть левые и правые векторные пространства. В наши дни под векторным пространством понимается исключительно модуль над полем, а эти объекты называются левый и правый модуль над телом. Поэтому лучше начать с какого-нибудь "Введения" Кострикина или с Винберга.
Аноним 07/11/15 Суб 17:57:20 #263 №309877 
>>309828
>Для бесконечных вроде 2x2 матрицы с жнф в качестве А.
Не все жнф матрицы 2x2 коммутируют.
Аноним 07/11/15 Суб 18:09:34 #264 №309878 
14469089748030.png
Является ли истинным высказывание "для любого $a \in A$ выполняется $P(a)$", если $A$ пусто? По логике да. Но вот, например, 4.2 на пикрелейтед. Возьмем в качестве пространства $X$ множество целых чисел с метрикой $d(x, y) = |x - y|$. Тогда в таком пространстве нет ни одной сходящейся к $p$ последовательности такой, что $\forall n: p_n \neq p$.

Есть подозрение, что тут подразумевается, что $X$ такого, что любая окрестность (в данном случае окрестность - это шар), содержит бесконечное число точек. Есть какое-то специальное название для такого пространства?что для любой точки существует последовательность, сходящаяся к этой точке.
Аноним 07/11/15 Суб 18:20:16 #265 №309879 
>>309878
На твоём скрине есть слово "limit point". В дискретной метрике нет предельных точек.
Аноним 07/11/15 Суб 18:29:39 #266 №309881 
А все, понятно. Ну да, если $p$ - предельная точка, то понятно, что существует последовательность, сходящаяся к $p$.
Аноним 07/11/15 Суб 18:32:26 #267 №309883 
>>309881
Ну и у которой $p_n \neq p$.
Аноним 07/11/15 Суб 18:42:33 #268 №309890 
14469109537130.jpg
>>309862

У меня 20, только что проверил.
Аноним 07/11/15 Суб 21:30:06 #269 №309910 
да, я знаю, что это очень легко решается и все такое, но, гайз, хелп ми плиз.
суть : нужно найти все такие значения параметра "а", что бы неравенство х^2-(3а-5)*х-а+(5/4) ≤ 0 , имело ровно 2 целых ответа, при этом оба отрицательных. Я понимаю, что это парабола, но на этом все.
Аноним 07/11/15 Суб 23:40:27 #270 №309924 
>>309373
Нет, конечно нет. Пусть $\alpha=\beta=\omega$ и $A_{\alpha,\beta}$ попарно различные одноэлементные множества. Тогда $$|\bigcup_{\beta \in K}\prod_{\alpha \in J} A_{\alpha,\beta}|=\aleph_0,$$ но $$|\prod_{\alpha \in J} \bigcup_{\beta \in K} A_{\alpha,\beta}|=2^{\aleph_0}.$$
Аноним 08/11/15 Вск 00:32:46 #271 №309925 
>>309924
Получается, я проебался.
Аноним 08/11/15 Вск 13:12:15 #272 №309966 
>>309910
Твоя парабола должна пересекать прямую x = 0. Т.е дискриминант больше 0. Дальше расстояние между корнями должно быть больше или равно 1, но меньше 3 - чтобы между ними влезло ровно 2 корня. Только это не достаточное условие, потом надо будет перепроверить. Наконец, оба корня должны быть отрицательными. Составляет систему из этих условий, решаешь, потом перепроверяешь.
Аноним 08/11/15 Вск 15:48:10 #273 №309983 
14469868902000.jpg
Поясните как из верхней формулы вышла нижняя.
Аноним 08/11/15 Вск 16:49:19 #274 №310001 
14469905592490.png
14469905592501.jpg
Аноним 08/11/15 Вск 16:58:13 #275 №310006 
>>309983
Сложением всех уравнений сверху. Нахуй ты этим картофаном занимаешься? Норм подход такой: нужно заметить, что формула для $sum k^p$ - это многочлен степени $p + 1$. Вычисляешь эту сумму для $n$ от $1$ до $p + 2$, решаешь систему линейных уравнений относительно коэффициентов многочлена.
Аноним 08/11/15 Вск 17:01:23 #276 №310007 
>>310006
>>309983
Кстати, это Спивак, да? Не читай эту хуйню, читай Рудина. Упражнения там по сложности такие же, но там хотя бы нет хуиллиарда ненужных упражнений и теория дается намного лучше.
Аноним 08/11/15 Вск 17:32:30 #277 №310014 
>>310006
>Сложением всех уравнений сверху.
Да, я не заметил там "and adding".
>>310007
Да ладно, я после Спивака думаю Марсдена/Тромбу навернуть (векторный анализ), а потом Матан Апостола.
Аноним 08/11/15 Вск 17:51:09 #278 №310018 
>>310001
И то верно, что это за учёный такой, без собственного бизнеса, кому он нужен.

>>310007
Давно хотел спросить, а диф.гем от спивака как? Такая же параша?

Аноним 08/11/15 Вск 21:57:05 #279 №310116 
>>310018
>Давно хотел спросить, а диф.гем от спивака как?
https://www.physicsforums.com/threads/a-comprehensive-introduction-to-differential-geometry-series-by-spivak.666556/
Аноним 08/11/15 Вск 23:17:39 #280 №310133 
Анонче, помоги. Нихрена не понимаю.
Составить уравнение геометрического места точек, отношение расстояний которых до данной точки А(10,0) и до прямой х=12,5 равны числу К=4/5. Полученное уравнение привести к каноническому виду и построить кривую.

Уравнение вышло в итоге такое: 9x^2+100x-25y^2=0
А что делать дальше, как приводить к каноническому виду и строить - не ебу. Сотни нефти тому, кто поможет.
Аноним 09/11/15 Пнд 01:26:44 #281 №310177 
14470216050180.png
Аноны, поясните, где на пикреле (если что, это Зорич) используется гладкость фи. Если следовать определению первообразной и доказывать прямым дифференцированием, то я не понимаю, где используется гладкость или зачем она там нужна. Я понимаю, что гладкость даёт непрерывность производной. Но зачем она здесь, если в определении от подынтегральной функции непрерывность не требуется, а для дифференцируемости композиции достаточно дифференцируемости обеих функций. Я понимаю, зачем требуется гладкость у определённого интеграла, но здесь что-то не могу уловить. Что я забыл?
Аноним 09/11/15 Пнд 05:28:27 #282 №310193 
>>310177
φ'(t)dt
Аноним 09/11/15 Пнд 13:35:10 #283 №310228 
>>309924
Хорошо. Тогда возникает такой вопрос. Пусть $X_\alpha$ - топологические пространства и $U$ - открытое подмножество $\prod_{\alpha\in J} X_\alpha$. Верно ли, что $U$ всегда имеет вид $U = \prod_{\alpha \in J} U_\alpha$, где $U_\alpha$ открыто в $X_\alpha$?
Аноним 09/11/15 Пнд 13:41:30 #284 №310230 
>>310228
Все, понял, что хуйню сказал. Можно взять обычный круг на плоскости и это уже будет не так.
Аноним 09/11/15 Пнд 18:34:01 #285 №310296 
>>310193
И что здесь даёт непрерывность? То, что можно будет написать dφ(t)? Так это и так можно будет сделать (хватит дифференцируемости). Можешь немного развернуть мысль?
Аноним 09/11/15 Пнд 19:21:45 #286 №310301 
Незадолго до сессии обнаружил что матан(тм) напрочь и успешно забыл.
Реквестирую способов за неделю или чуть больше ликвидировать неграмотность в школьном/первосеместровом калькулюсе.
Этого времени должно хватить чтобы вспомнить элементарные вещи, у меня все-таки не совсем нулевой левел.
Ключевые слова: непрерывность, ряды, сходимость, формула тейлора.
Толстые книги типа томов Зорича к сожалению не подойдут (нет времени), но если скажете конкретные главы/места которых должно хватить буду очень благодарен.
knuebok 09/11/15 Пнд 19:56:40 #287 №310306 
>>310177
Она не нужна, ты всё правильно понял.
Аноним 09/11/15 Пнд 20:03:37 #288 №310307 
>>310306
Да, спасибо, я уже создал тему на dxdy. Сказали то же самое. Всё-таки иногда Зорич меня вводит в ступор. Я привык, что в математике стараются использовать как можно более слабые определения и как можно более общие выводы, но, когда без какой-либо мотивировки вводится сильное утверждение взамен слабого, я впадаю в ступор и начинаю нервничать, ибо возможны два варианта: 1) автор по наитию так сделал; 2) я что-то забыл или не понимаю. Наверно, мне не хватает уверенности.
Аноним 09/11/15 Пнд 20:07:00 #289 №310308 
>>310301
http://www.maths.ed.ac.uk/~aar/papers/maclanecat.pdf
подойдёт
knuebok 09/11/15 Пнд 20:16:00 #290 №310310 
>Я привык, что в математике стараются использовать как можно более слабые определения и как можно более общие выводы
Ну это не всегда так, далеко не всегда. Кстати, вот насчёт "дифференцируемых, но не непрерывно дифференцируемых функций" - пример хороший очень, дело в том, что никаких техник для тонкой работы с такими функциями нету. Вообще, если пример построен не искуственно, то никаких методов для доказательства того, что f - дифференцируемая, не доказывая при этом то, что f - непрерывно дифференцируема, кажется, тоже нету, об этом даже Тао говорил. А если техник для работы с ними нету, то зачем вообще на них внимание заострять? Никогда не задумывался, почему для множества "непрерывно-дифференцируемых функций на Х" ввели значок C^(1) (X), а для множества "просто дифференцируемых функций на Х" никакого общепринятого значка нету?
>Всё-таки иногда Зорич меня вводит в ступор.
Да у него много там "приколов" таких, когда он пытается наводить максимальную строгость, но прокалывается в некоторых тонких местах. Но мне кажется это не смертельно, лично я этот учебник люблю безмерно, как мой первый учебник по труъ математике, и всем его советую.
Аноним 09/11/15 Пнд 20:20:00 #291 №310312 
>>310308
мимо твой мемос, категориями я итак упарываюсь (хотя этот учебник кажется довольно уродливым)
мне бы теперь дифурчики и интегралы научиться брать
Аноним 09/11/15 Пнд 20:34:13 #292 №310314 
>>310310
>А если техник для работы с ними нету, то зачем вообще на них внимание заострять?
Ну да, кажется, я понял. Дифференцируемость функции слишком общий случай, из которого «относительно мало» полезных свойств можно изъять. Так же как полугруппу наделают дополнительными свойствами и изучают её уже как группу. Хотя в том примере это не так уж важно. Главное, установить равенство, а чем более общий случай равенства мы можем установить, тем лучше.

>а для множества "просто дифференцируемых функций на Х" никакого общепринятого значка нету?
Видимо, потому что слишком общий случай, чтобы его выделять. Из дифференцируемости функции многого о ней не скажешь.

> лично я этот учебник люблю безмерно
Мне он тоже очень нравится, поэтому и читаю. Не всё гладко, конечно, но я могу сравнить только с устаревшим Фихтенгольцем (в лучшую сторону, естественно). А сколько времени, если не секрет, у тебя ушло на его чтение? Ну чтобы прямо так основательно, впитать каждую букву, а не просто пролистать. Да я тот анон, что спрашивал выше про чтение учебников.
knuebok 09/11/15 Пнд 21:01:51 #293 №310316 
>>310314
Написал длинный пост, но случайно резетнул комп 3: Если коротко: то потратил я на учебник год, но только потому, что занятия математикой были бессистемны и "под настроение", а ещё я из-за своего перфекционизма зачем-то латехал решения всех упражнений, лил. Насколько эффективно для образования читать въедливо каждую букву не очень понятно. Оказалось, в итоге, что в Зориче математику нужно далеко не всё: в частности, по интегралу Римана можешь сильно не задрачиваться, всё равно в итоге придётся потом учить (если продолжишь занятия математикой) более простой и концептуально-правильный интеграл Лебега. Я бы на твоём месте почитывал бы параллельно учебник "Львовский "Лекции по математическому анализу"", не въедливо, но хотя бы просто по диагонали, чтобы узнать о том, как можно строить анализ по-другому, да и вообще некоторые вещи там интересны довольно. Таки дела.
Аноним 09/11/15 Пнд 21:27:14 #294 №310319 
Аноны, помогите с решением уравнения (x-2)(x-3)(x+4)(x+5)=1320 готовлюсь к олимпиаде (10 класс)
Используя метод Феррари, бета у меня получается равной -76, а в педевикии рассматривается только решение при бета = 0. Открыл первую страницу с задачами и, мягко говоря, охуел с их сложности, ведь такого в программе не было точно. Алсо, порекомендуйте учебник по алгебре на уровень 10-11 классов
Нид хэлп, срочно
Аноним 09/11/15 Пнд 21:28:48 #295 №310320 
>>310316
>Насколько эффективно для образования читать въедливо каждую букву не очень понятно.
Я это делаю потому, что не могу нормально двигаться в теории, пока не буду убеждён в обоснованности каждого действия, даже если интуитивно мне абсолютно всё понятно.

>по интегралу Римана можешь сильно не задрачиваться
Да я не задрачиваюсь, скорее изучаю, потому что до недавнего времени всё, что я знал о слове интеграл, было: «Ну это тип площадь под графиком.» Конечно, когда видишь чоткое определение, то многие теоремы, факты и свойства практически сразу становятся очевидными.

>всё равно в итоге придётся потом учить (если продолжишь занятия математикой) более простой и концептуально-правильный интеграл Лебега
Да, уже знаком с ним (примерно на том же уровне, что и с интегралом Римана до недавнего времени). Одно из первых действий, которые я сделал, взяв учебник Зорича, -- это поиск упоминания интеграла Лебега. Как оказалось, в Зориче его это как-то всё вскользь, но это не так уж проблематично.

За Львовского спасибо. Действительно очень занимательно.
Аноним 09/11/15 Пнд 21:32:38 #296 №310321 
>>310319
Какой метод Феррари, ты поехал что ли? А если я дам тебе уравнение 5-й степени, что ты делать будешь?
Здесь совсем иной подход к решению задачи, она же "олимпиадная", а значит на соображалку. Хинт: разложи 1320 на простые множители.
Аноним 09/11/15 Пнд 21:34:41 #297 №310322 
>>310321
Я не сильно могу в алгебрухоть и отличник, что вообще не показатель, и поэтому просто начал гуглить по уравнениям четвёртой степени, и наткнулся на это. А с разложением надо попробовать, да
Аноним 09/11/15 Пнд 21:54:32 #298 №310328 
>>310321
Решил, всё нормально, спасибо за совет. Но разве нет другого выхода, кроме как подбирать корни?
Аноним 09/11/15 Пнд 22:10:11 #299 №310334 
>>310328
Я думаю это самый оптимальный метод. Неужели подбор корней занимает так много времени? По-моему корень 6 вылазит прям сразу после разложения. Второй — после некоторых размышлений, которые так же не занимают много времени.
Аноним 09/11/15 Пнд 22:11:59 #300 №310335 
>>310320

> Я это делаю потому, что не могу нормально двигаться в теории, пока не буду убеждён в обоснованности каждого действия, даже если интуитивно мне абсолютно всё понятно.


Тогда хотелось бы услышать как ты определяешь натуральное число???
Аноним 09/11/15 Пнд 22:15:03 #301 №310337 
>>310335
>этот отступ
N-петух, скажи, а какая всё-таки у тебя зарплата? Большая?
Аноним 09/11/15 Пнд 22:18:01 #302 №310338 
>>310337

Чуть выше средней по городу.
Аноним 09/11/15 Пнд 22:25:34 #303 №310342 
>>310335
Я бы не хотел участвовать в этой демагогии просто потому, что не обладаю достаточными знаниями. Ну скажем, в Зориче даётся такое определение:
«Множество натуральных чисел — это наименьшее индуктивное множество, содержащее 1, т.е. пересечение всех индуктивных множеств, содержащих число 1.»
Как тебе такое определение?
Аноним 09/11/15 Пнд 22:27:39 #304 №310343 
>>310342
>индуктивное
Пойду надену маску для защиты от жопной гари.
Аноним 09/11/15 Пнд 22:34:11 #305 №310353 
>>310343
>индуктивное
Множество, которое содержит пустое множество и последователь любого своего элемента. По аксиоме бесконечности такие множества существуют.
Аноним 09/11/15 Пнд 23:16:36 #306 №310384 
>>310352
>>310363
N-петух помимо вербального засирания информационного пространства треда перешёл ещё и к визуальному? Чую скоро моча тебя банхаммером ёбнет и придётся тебе разбираться с натуральными числами самому.
Аноним 10/11/15 Втр 01:11:46 #307 №310411 
>>310337
Это что это? Больше десяти зёрнышек?
Аноним 10/11/15 Втр 01:13:10 #308 №310412 
>>310312
Matematiku neobyazatelno sovershenno znat classicheskogo analiza.
Аноним 10/11/15 Втр 01:16:38 #309 №310413 
>>310412
Ti voobshe ne matematik sooq, huli ti tut filosofstvuesh ))
Аноним 10/11/15 Втр 09:37:37 #310 №310426 
>>309552

Оно не является самостоятельным, ты не можешь осуществить его (индуктивного множества) построение без опоры на какую нибудь форму (интуитивной) индукции. Точно так же при описании формальной системы тебе требуется какая-то форма индукции (хотя конечно можно строить их независимо, но тогда придется предполагать исходными два вида индукции, что будет не нужным усложнением). Т.е. в этом смысле оба эти понятия основаны на какой-то интуитивной индукции, я в примере выше и обозначил ее С, а до этого называл метаиндукцией. Синонимичность в данном случае я не требую, это бессмысленно так как можно всегда искусственно разделить термины через какой нибудь частный случай или соглашение (типа одни считают N начинается с 0, другие с 1 или пример разделения на множества и классы). Но что действительно важно, это то что все эти формы индукции по сути эквивалентны.
Аноним 10/11/15 Втр 13:24:54 #311 №310464 
>>310384
Уже ёбнула?
Аноним 10/11/15 Втр 13:44:45 #312 №310470 
>>310413
ti che sooka ti che blyad tebe moi h-index pokasat? moi lekcii po algemu suka?! da menya sam shen uchel poka ti pod stol peshkom sral blyad! da ti uyebok ot prirodi suka pizdec ti menya vibesil chtoby zavtra vo dvore IUM bil suka v 18:00 posle seminara!
Аноним 10/11/15 Втр 14:49:58 #313 №310481 
>>310464

Твою мамку-шлюху ёбнули.
Аноним 10/11/15 Втр 15:03:51 #314 №310497 
чем {0;2} отличается от [0;2]?
Аноним 10/11/15 Втр 15:21:26 #315 №310507 
>>310497
Мм, всем.
Аноним 10/11/15 Втр 15:21:47 #316 №310508 
>>310507
спасибо, я уже понял, извините за тупой вопрос
Аноним 10/11/15 Втр 16:15:01 #317 №310533 
>>310426
>Т.е. в этом смысле оба эти понятия основаны на какой-то интуитивной индукции, я в примере выше и обозначил ее С, а до этого называл метаиндукцией.
Согласен. Это в принципе то же что и я говорю:
>То что оба этих объекта так или иначе "относятся" к общему понятию "индукция"
Но вместо избыточного термина "метаиндукция", я использую общее понятие индукции. Но:
>не значит то что они ссылаются друг на друга.
И:
>Таким образом, цикличности я здесь не вижу.
Аноним 10/11/15 Втр 18:29:50 #318 №310581 
Братишки нужна ваша помощь.
1) линеен ли оператор Ax(t)=sin(tx1) A:R^2 -> C(R)
2) придумать оператор A:C[0,1] -> C[0, +бесконечность)
knuebok 10/11/15 Втр 19:24:43 #319 №310601 
>Ax(t)=sin(tx1) A:R^2 -> C(R)
Я не очень понял, у тебя А зависит от двух параметров, однако, а формула t -> sin(t x) от одного.
knuebok 10/11/15 Втр 19:31:04 #320 №310608 
>2) придумать оператор A:C[0,1] -> C[0, +бесконечность)
0
Аноним 10/11/15 Втр 19:46:17 #321 №310616 
>>310601
в смысле оператор а применяется к функции x(t) sin(tx1) здесь t её аргумент, а x1 это первая координата точки R2 же
Аноним 10/11/15 Втр 19:50:05 #322 №310619 
>>310616
Значение x(t) принадлежит R2
Аноним 10/11/15 Втр 19:50:52 #323 №310620 
>>310619
>Значение x(t) принадлежит R2?
Аноним 10/11/15 Втр 20:01:13 #324 №310623 
>>310620
x(t) задана на плоскости
Аноним 10/11/15 Втр 20:06:15 #325 №310627 
>>310623
Так, процитируй условие задачи. Иначе тут разговор слепого с глухим получается.
С одной стороны A:R2 → C(R)
С другой: Ax(t) = sin(tx1) где 1 первая координата точки в R2.
Что это за точка? Это значение x(t) или нет?
Что значит t? Это одна переменная или вектор?

Пока условие неясно.
Аноним 10/11/15 Втр 20:11:08 #326 №310631 
>>310627
если x(t) задана в R2 значит t это вектор и t=(x1, x2)
В том то и дело, что условие задачи именно такое как я написал его сначала, препод дал задачи и нихера не объяснил, что есть что
Аноним 10/11/15 Втр 20:47:58 #327 №310645 
>>310631
> препод дал задачи и нихера не объяснил, что есть что
Видимо, он не думал, что ты даун и не можешь спросить, какая буква что означает?
Аноним 10/11/15 Втр 20:53:46 #328 №310649 
>>310645
>если x(t) задана в R2 значит t это вектор и t=(x1, x2)
это на иврите написано или на каком-то еще, неизвестном тебе, языке
Аноним 10/11/15 Втр 21:21:27 #329 №310667 
>>309844
Хватит обсуждать чью-то домашку. Лучше на >>309844 ответьте.
Аноним 10/11/15 Втр 21:30:56 #330 №310669 
>>309844
Функциональный интеграл, он же интеграл по путям
Аноним 10/11/15 Втр 23:33:37 #331 №310678 
>>310631
Ну, короче, синус в любом из возможных случаев вряд ли линеен, так что отвечай так: "Нет, не линеен".
Аноним 10/11/15 Втр 23:52:22 #332 №310679 
>>310677
иди нахуй со своим картофелем
Аноним 11/11/15 Срд 00:00:33 #333 №310681 
>>310679
>иди нахуй со своим картофелем
Ну позязя, анончик, я весь вечер провозился с этой хуйнюшкой.
Аноним 11/11/15 Срд 01:35:22 #334 №310688 
14471949225860.png
>>310677
В ряд раскладывай все, что видишь.
Аноним 11/11/15 Срд 01:50:40 #335 №310692 
>>310470
Каледин, залогинься.
Аноним 11/11/15 Срд 02:15:11 #336 №310693 
14471973117100.jpg
>>310688
Аноним 11/11/15 Срд 02:27:50 #337 №310694 
>>310470
Забавно потому что вербицкий и подобные (как и любые матшкольники) все эти основы анализа, вплоть до простых дифуров и интегралов, узнали еще в старших классах
Аноним 11/11/15 Срд 06:34:24 #338 №310696 
14472128649130.jpg
>>310694
Аноним 11/11/15 Срд 18:12:52 #339 №310801 
>>310321
>она же "олимпиадная", а значит на стандартные методы решения "олимпиадных" задач
Пофиксил этого четвертьфиналиста.
Аноним 11/11/15 Срд 18:22:46 #340 №310805 
>>310801
Всегда проигрываю с этих дебилов, для которых операции уровня разложить четырехзначное число на множители и решить пару системок линейных уравнений это ОЛИМПИАДКИ СМЕКАЛОЧКА КОКОКО НИНУЖНО НАРМАЛЬНЫИ МАТИМАТИКИ ТАКОВА НИ ДАЛЖНЫ УМЕТЬ
Аноним 11/11/15 Срд 18:29:25 #341 №310808 
>>310801
двачую этого
>>310805
в олимпиадных задачах нет ничего сверхъестественно олимпиадного, у составителей смекалочка не ахти как работает поэтому они просто юзают материал для следующего класса при составлении задач и не парятся. Ну по крайней мере так было, просто для проформы добавил.
Аноним 11/11/15 Срд 18:30:13 #342 №310809 
>>310319
>Открыл первую страницу с задачами
Че за книга? Можешь скинуть?
Аноним 11/11/15 Срд 18:49:26 #343 №310817 
>>310808
Ну вот ты и сам сказал, что в олимпиадных задачах нет ничего именно олимпиадного, т.е. отличного по духу от всего остального и неприменимого больше нигде. В учебничках к теме дают задачки - там нужно поюзать метод из темы, плюс немножко из пройденного. Задачки со звездочкой оттуда же - это помимо юзания метода еще и придумать, как его юзнуть. Олимпиадные задачки - это следующий этап после задачек со звездочкой: надо самому выбрать, какие методы юзать, и как. Этим же самым любой математик будет заниматься хотя бы при написании диссера - выбором методов и придумыванием, как и на чем их юзнуть, чтобы получить результат. Просто задачи будут глобальнее и методов будет больше, но один хуй этому учиться придется, если хочется чего-то достичь. Ну а неспособность осилить методы школьной/вузовской программы или пользоваться ими (что однохуйственно) сразу обо всем говорит.
Про материал для следующего класса ты совершенно не прав, между прочим. Задачи даются на ту школьную теорию, которая школьником такого-то класса пройдена. С затрагиванием вещей, которые обычным школьникам вообще не преподают - но в такой форме и в таком размере, чтобы они смогли это понять и решить.
Аноним 11/11/15 Срд 19:33:30 #344 №310833 
>>310817
>Про материал для следующего класса ты совершенно не прав, между прочим.
В теории давать задачи, требующие материал следующего класса, не комильфо. Но на самом деле все олимпиадники знают материал следующего класса. В этом соль олимпиадного ракования.
Аноним 11/11/15 Срд 20:32:33 #345 №310857 
>>310833
В соревнованихя по бегу в мешках побеждает не тот кто умеет хорошо бегать, а тот кто умеет хорошо бегать в мешках. В этом суть олимпиадного ракования.
Аноним 11/11/15 Срд 21:06:00 #346 №310868 
>>310310
>лично я этот учебник люблю безмерно, как мой первый учебник по труъ математике, и всем его советую.
Пиздец. Нахуй так жить.
Аноним 11/11/15 Срд 21:18:55 #347 №310869 
>>310857
В этом суть любого соревнования вообще: лучше в нем себя показывает тот, кто к нему готовился, чем тот, кто не готовился. ЕГЭ пишут лучше те, кто решал задачи для ЕГЭ, чем просто дохуя умный (потому что умный может где-нибудь обосраться со знаком, например). Новые результаты в области больше открывают поработавшие в этой области профессора, нежели просто залетные мамкины гении. Что сказать-то хотел?
Аноним 11/11/15 Срд 21:24:22 #348 №310871 
>>310868
Посоветуешь что-то лучше?
Аноним 11/11/15 Срд 21:35:47 #349 №310875 
>>310869
Нихуя они не открывают, это старое мудилы, "пробежать быстрее" - не значит "открыть новую дорогу".
То, что наверное анон хотел сказать - что ваши олимпиады имеют мало отношения к жизни реальной, когда во-первых тебе нередко нужно разбираться в хуйне, которую ты в первый раз видишь, а во-вторых проблемы часто вообще лежат в ненаучном русле, но задроты, коими является большинство "ученых" предпочитают вообще эту сферу игнорировать.
Аноним 11/11/15 Срд 22:06:12 #350 №310878 
>>308111 (OP)
Есть ли символ, или как мне быть с разностью в количество раз между двумя числами? То есть, модуль разности двух чисел - их скалярная разность, на сколько два числа отличаются друг от друга, а вот что же сделать с двумя числами, которые не можешь просто разделить, так как не знаешь, кое из них наибольшее?
Аноним 11/11/15 Срд 22:34:16 #351 №310879 
Посоветуйте учебник, где пошагово и полно излагается теория групп. Меня интересуют, в частности, группы Ли.
Никак не могу определиться, что по этому вопросу почитать.
Аноним 11/11/15 Срд 22:38:44 #352 №310881 
>>310879
Выудил из этого треда учебник вавилова, так что остаётся только вторая половина вопроса - насчёт групп Ли.
Аноним 11/11/15 Срд 23:53:20 #353 №310903 
>>310881
http://math.hse.ru/books
Аноним 12/11/15 Чтв 00:00:16 #354 №310904 
>>310878
Переформулируй вопрос.
Аноним 12/11/15 Чтв 01:36:04 #355 №310916 
Есть диффур du/dt + λu(t) + iwu(t) = eE/m, где u=dx/dt + i dy/dt, решением которого является функция u(t) = eE/m(λ+iw) + Cexp(-λt-iwt). Начальные условия (dx/dt, dy/dt) = (0, 0). Как найти здесь константу? Если взять u(t) = 0 и приравнять, то в итоге хуйня полная получается и зависимость от t пропадает.
Аноним 12/11/15 Чтв 08:36:52 #356 №310959 
>>310916
>u(t) = eE/m(λ+iw) + Cexp(-λt-iwt)
0 = eE/m(λ+iw) + C
что тут не так? вроде норм, отсюда константа находится
Аноним 12/11/15 Чтв 11:48:10 #357 №310984 
Как сгенерировать все простые (без петель и кратных ребер) графы, состоящие из $n$, вершин с точностью до изоморфизма?
Аноним 12/11/15 Чтв 12:52:42 #358 №310994 
Господа, а поясните пожалуйста доходчиво, почему критикуется эпсилон-дельта формализм?
Аноним 12/11/15 Чтв 13:38:41 #359 №310999 
>>310994
В нём используется метаиндукция.
Аноним 12/11/15 Чтв 15:07:42 #360 №311013 
>>310994
Т.к. он всегда или почти всегда может быть заменен на более общий топологический.
Аноним 12/11/15 Чтв 15:10:08 #361 №311014 
>>310994
Он труден для восприятия, неудобен в работе и может быть заменён более удобным и более общим формализмом фильтров.
Аноним 12/11/15 Чтв 16:23:24 #362 №311035 
>>310994
Ну ты можешь считать, что окрестность точки p - это любое открытое множество, содержащее p (а не только шар, как в метрическом пространстве). Я не особо шарю, но мне кажется, есть смысл изучать топологию, если ты собираешься изучать какие-то неметризуемые топологические пространства (http://mathoverflow.net/questions/52032/examples-of-non-metrizable-spaces) либо просто из интереса. Метрическое пространство намного понятнее топологического и геометрическая интуиция там работает.

Аноним 12/11/15 Чтв 16:31:57 #363 №311038 
14473351175010.png
Увидел N-петуха в /pr/ и проиграл в голосяндру.
Аноним 12/11/15 Чтв 17:04:40 #364 №311048 
>>311038
Ну так обосравшись тут он пошел в другие разделы
Аноним 12/11/15 Чтв 17:39:29 #365 №311052 
>>311048
> обосравшись тут
Так N-петух приводил нормальные аргументы, просто определение N интересно только узкому кругу опущенцев.
Аноним 12/11/15 Чтв 18:15:46 #366 №311057 
>>311052
Его аргументация сводилась к тому, что в определении N при помощи аксиом Пеано - присутствует цикличность. Неверно (видимо в силу недостатка знаний и опыта) интерпретируя понятия "индуктивное множество", "аксиома индукции" и оперируя общим понятием "индукция" (почему-то заменив его на метаиндукцию), он утверждал о цикличности.

В диалоге с ним: >>307919, я пытался показать, что цикличности здесь нет.
Аноним 12/11/15 Чтв 18:16:40 #367 №311058 
>>311038
Да похуй, главное что не здесь.
Аноним 12/11/15 Чтв 18:17:33 #368 №311059 
14473414537340.jpg
>>311057>>311052>>311048>>311038
>N-петух
Полчаса в пейнте, и он с нами!
Аноним 12/11/15 Чтв 18:19:07 #369 №311060 
>>311038
Няш, доставь ему в тред в /pr/ вот это, пусть порадуется: >>311059
Аноним 12/11/15 Чтв 19:09:04 #370 №311065 
господа, подскажите годную литературу по теории игр
с меня печеньки
Аноним 12/11/15 Чтв 20:17:58 #371 №311071 
>>311057
Ты до сих пор думаешь, что он искренне заблуждается, а не просто толсто троллирует?
Аноним 12/11/15 Чтв 20:56:00 #372 №311077 
>>311065
Читай классику D&D
Аноним 12/11/15 Чтв 20:59:01 #373 №311078 
>>311059
10/10
Аноним 12/11/15 Чтв 21:04:03 #374 №311079 
14473514439540.jpg
14473514439571.jpg
Наткнулся на сайт странного человека под именем Barry Dalgarno, и он либо поехавший, либо самый гениальный человек в мире. Пишет, что нашел закономерность в простых числах и даже как то ее описывает.

Основной текст: https://sites.google.com/site/geometryoftheprimes/
Карта сайта (сайт большой, навигация в принципе отсутствует) https://sites.google.com/site/geometryoftheprimes/system/app/pages/recentChanges

Картинки у него уж слишком красивые получаются, получше спирали Вильяма, но алгоритм построения лично я так и не понял. Возможно, просто я тупой.
Аноним 12/11/15 Чтв 21:18:32 #375 №311083 
>>311079
Серега Герасимов, опять ты?
Аноним 12/11/15 Чтв 21:19:50 #376 №311084 
>>311083
Не знаю таких, кто это?
Аноним 12/11/15 Чтв 21:38:44 #377 №311090 
14473535247860.png
>>311079
Такой гениальный, а в тех так и не научился.
Аноним 12/11/15 Чтв 21:40:16 #378 №311091 
>>311071
Скорее всего. Но даже если это и троллинг, то своей цели он не достиг. Цель троллинга - раздражение, провоцирование агрессии. В процессе объяснения её не было (а это раздражает тролля). Это одна из тактик т.н. "backtrolling"-а.
Аноним 12/11/15 Чтв 22:04:46 #379 №311102 
>>311090
Не задевайтесь словами. Что там по сути то?
Аноним 12/11/15 Чтв 22:23:33 #380 №311109 
>>310904
|5-3| = |3-5|
волшебныйсимвол5/3волшебныйсимвол = волшебныйсимвол3/5волшебныйсимвол
Что за символ?
Аноним 12/11/15 Чтв 23:38:28 #381 №311127 
>>309844
>А есть ли бесконечно-кратный интеграл?
Естественно, блядь. Определи его как предел последовательности.

мимо-любитель-картофанчика
Аноним 12/11/15 Чтв 23:45:16 #382 №311129 
>>311109
Ты в "что где когда" играешь?
Аноним 12/11/15 Чтв 23:50:14 #383 №311130 
>>311038
Лол, случайно зашёл в раздел в случайный тред и увидел свою руками переписанную копипасту аксиом Пеано.

Раз уж так сложилось, помогите мне: что у нас есть из дискретных ортогональных базисов, кроме Хаара (вейвлет) и Уолша-Адамара?

Формулировка задачи такая: у меня есть матрица n X n значений от 0 до n-1. n-значная логика от 2 переменных, говоря языком дискретки. Смотрю визуализацию этого дела и чую, что описать можно как-то просто. Но с Хааром и Уолшем-Адамаром обосрамшись, почти все коэффициенты не равны нулю и вообще выглядят как шум.
Аноним 13/11/15 Птн 00:25:21 #384 №311137 
Сап, посоны. Ищу годные книги по формальной логике и математической логике. Был бы благодарен, если кто-то посоветует годную литру.
Аноним 13/11/15 Птн 00:35:06 #385 №311138 
>>311130
Вот есть идея: Обобщением преобразования Уолша является преобразование Виленкина — Крестенсона:

Используется при анализе и синтезе устройств автоматики с элементами, выполняющими операции троичной и q - ичной логики.

У меня как раз n-значная логика...

Кто-нибудь сталкивался?
Аноним 13/11/15 Птн 01:54:52 #386 №311147 
>>311137
Текущий уровень знаний логики и математической культуры в целом (школьникам и аспирантам математикам подходят совсем разные книги)?
Аноним 13/11/15 Птн 02:00:43 #387 №311148 
>>311137
> формальной логике и математической логике
Это по сути синонимы. А по теме, гугли книгу Э. Мендельсон Мат. логика
Аноним 13/11/15 Птн 02:22:57 #388 №311151 
>>311148
Я бы не рекомендовал. В качестве доступного учебника Верещагин Шень куда удачнее. На роль учебника более продвинутого уровня куда лучше подходит Шенфилд.
Аноним 13/11/15 Птн 04:47:05 #389 №311156 
>>311079
Всем похуй?
Аноним 13/11/15 Птн 05:13:18 #390 №311157 
14473807985610.gif
14473807985621.jpg
14473807985632.jpg
Есть ли математическое название/описание построения похожих листоподобных бесконечных фракталов?
Аноним 13/11/15 Птн 05:16:01 #391 №311158 
>>311157
Чтобы было понятнее, требуется это для построения бесконечной улицы. Поэтому особая "природность" не нужна.
Аноним 13/11/15 Птн 07:57:14 #392 №311163 
>>311079 >>311156
Мне не похуй. Сразу же спиздил ссылку себе в текстовичок. Смущает то, что у него математический аппарат подробно не расписан. Я привык чтоб разжёвано было, а тут просто что-то типа Фибоначчи графически.

Где откопать матаппарат к картиночкам?
Аноним 13/11/15 Птн 08:45:36 #393 №311165 
>>311127
Можешь показать?
Аноним 13/11/15 Птн 09:14:54 #394 №311173 
>>311151

> Верещагин Шень куда удачнее.

Тупое говно которым даже подтереться нельзя так как оно в электронном виде.
Аноним 13/11/15 Птн 09:33:25 #395 №311176 
>>311165
Ну посмотри в любом учебнике, как определяется предел последовательности. Вот у тебя будет предел последовательностей рекурсивного взятия кратного интеграла. Даже задачку по матану такую помню, посчитать многомерный кратный интеграл.
Аноним 13/11/15 Птн 09:35:27 #396 №311179 
>>311057

Ты признал что в обоих случаях имеет место прямая зависимость от интуитивного понятия индукции, т.е. без нее данные понятия не существуют так как их попросту нельзя сконструировать.

Насчёт цикличности к которой ты прикопался, ок я соглашусь с тобой так как это абсолютно не принципиально в той формулировке которую ты используешь (я по старой памяти думал ты понимаешь аксиоматику Пеано в кнуёбковском смысле, т.е. как реальное определение N "из ничего").



Аноним 13/11/15 Птн 09:48:41 #397 №311180 
>>311176
Но ведь интеграл не обязан быть счётнократным. А предел по Гейне сработает максимум для континуально-кратного интеграла.
Аноним 13/11/15 Птн 09:53:26 #398 №311181 
>>311057

> Его аргументация сводилась к тому, что в определении N при помощи аксиом Пеано - присутствует цикличность.

Цикличность понятий не означает что они абсолютно тождественны, если пример с множествами не нравится, то смотри школьное определение точки например.

Я имел ввиду неустранимую зависимость от интуитивной индукции поскольку она является сутью натурального ряда, все остальное бантики.
Аноним 13/11/15 Птн 11:15:11 #399 №311188 
14474025117030.png
Сап, хочу распознавать цифры. Вот отсюда я взял датасет:
https://www.kaggle.com/c/digit-recognizer/data
Если лень читать, то картинка представляет собой матрицу 28*28 из чисел от 0 до 255; чем темнее пиксель, тем больше число на соответствующей позиции.

Я решил сначала научиться отвечать на вопрос, является данная цифра единицей, или это что-то другое. Самое банальное решение: выход - линейная функция от всех входов, то есть каждый пиксель имеет какой-то вес $w_i$. Пытаюсь сделать градиентный спуск, но не получается. Формулу градиентного спуска я вывел следующим образом. Пусть у нас training set $T$ состоит из пар $(x, t)$, где $x$ - вход и $t$ - выход, который должен получиться. Воспользуемся методом наименьших квадратов. Суммарная ошибка равна
$$E = \frac{1}{2}\sum_{(x, t) \in T} (w^Tx - t)^2$$
$$\frac{\partial E}{\partial w_k} = \sum_{(x, t) \in T} (w^Tx - t)\frac{\partial(w^Tx - t)}{\partial w_k} = \sum_{(x, t) \in T} (w^Tx - t) x_k$$
Тогда на каждом шаге нужно изменять $w$ следующим образом:
$$w = w - \eta \sum_{(x, t) \in T} (w^Tx - t) x,$$
где для $n$-й итерации $\eta = с / n$. Я пробовал подбирать разные значения $c$, но проблема явно не в этом.

Вот код:
http://pastebin.com/F0g93bHF

Почему-то у меня некоторые веса становятся порядка 10^60. Я неправильно вывел формулу, или эту задачу нельзя решить градиентным спуском?
Аноним 13/11/15 Птн 11:59:36 #400 №311190 
>>311180
Я то ли в маразм впал, то ли не понимаю твоих трудностей.

a_n при n-> +inf

где n -- кол-во кратностей интеграла. Счётное, как и нужно
Аноним 13/11/15 Птн 12:02:40 #401 №311192 
>>311190
Ну и рекурсивно задаётся a_n = _/ a_{n-1} dx
Аноним 13/11/15 Птн 12:22:46 #402 №311195 
>>311190
Пусть n - какой-то ординал.
Нужно определить ∫n. Если n - конечный ординал, то всё очевидно. Если n = ω, то интеграл очевидно определяется так, как ты сказал. Но как быть, если n > ω? У меня есть подозрение, что теория меры меня обломает.
Аноним 13/11/15 Птн 12:24:52 #403 №311196 
>>311173
В нем обходятся многие тонкости, но ровно это и делает его хорошим учебником для людей с низким уровнем предварительной подготовки. Кстати, он несколько раз переиздавался и его должно быть легко купить.
Аноним 13/11/15 Птн 12:29:34 #404 №311197 
>>311158
Вообще, похоже тебе нужно то, что обычно называется замощение, только с фракталами.
Вот из первых ссылок, что выдаёт по этому запросу:
https://en.wikipedia.org/wiki/Pinwheel_tiling
https://en.wikipedia.org/wiki/Aperiodic_tiling
Покопай в эту сторону.
Аноним 13/11/15 Птн 14:10:40 #405 №311212 
>>311157
L-systems же.
Аноним 13/11/15 Птн 14:11:24 #406 №311213 
>>311188
Все пацаны, отбой, ошибка была в коде.
Аноним 13/11/15 Птн 14:32:22 #407 №311216 
>>311196

Какие тонкости?
Аноним 13/11/15 Птн 14:56:24 #408 №311224 
>>311179
>Ты признал что в обоих случаях имеет место прямая зависимость от интуитивного понятия индукции, т.е. без нее данные понятия не существуют так как их попросту нельзя сконструировать.
Не знаю что ты называешь "прямой зависимостью", но без интуитивного понимания индукции, на одной формализации действительно далеко не уедешь. Также как без интуитивного понимания точки или предела последовательности. Но строгость в математике требует формального определения понятий, и оно было дано в логике второго порядка.

>Насчёт цикличности к которой ты прикопался, ок я соглашусь с тобой
Так.

>так как это абсолютно не принципиально в той формулировке которую ты используешь
Не принципиально для чего?

>Я имел ввиду неустранимую зависимость от интуитивной индукции
Так.

>поскольку она является сутью натурального ряда,
Что это значит?

>все остальное бантики.
Ты извини, но как известно "the devil in the details". Если ты формальную систему построенную как индуктивное множество, в которой присутствует аксиома индукции, если ты всю эту конструкцию несущую определённую информацию в областях математики и играющую важную роль в доказательствах называешь "бантиками", то это лишь показывает, что ты не видишь значения определения, и не ориентируешься в этих областях, пытаясь только интуитивно понять этот объект и попадая в ловушку собственной интуиции, ошибочно утверждая о "замкнутом круге".
Аноним 13/11/15 Птн 15:30:50 #409 №311234 
>>311224

> Не знаю что ты называешь "прямой зависимостью"

Я же сразу после ближайшей запятой пояснил что я имею ввиду.

> но без интуитивного понимания индукции, на одной формализации действительно далеко не уедешь.

До формализма ты даже не дойдешь, ты его без индукции построить не сможешь.

> Также как без интуитивного понимания точки или предела последовательности.

Ты похоже вообще ничего не понял. Речь не про интуитивные образы помогающие понять или найти решение, а про исходные понятия без которых невозможно сконструировать простейшие вещи вроде формальной системы. Для геометрии как раз интуитивного представления о точке не требуется если есть система формальных аксиом. Как говорил Гильберт вместо точек могут быть пивные бутылки. Для натуральных чисел так не получится потому, что избавиться от интуитивного представления об индукции при построении аксиоматики Пеано невозможно.

> и оно было дано в логике второго порядка.

Логика второго порядка требует пререквизита в виде индукции.

> Не принципиально для чего?

Для того чтобы утверждать об неустранимости интуитивной индукции на которой всё строится (меня устраивает такая более слабая формулировка). Иначе ты будешь пытаться отрицать наличие цикличности на основе каких-то своих представления о ней.

> Что это значит?

Натуральный ряд это какой-то единичный объект к которому применена индукция. Первое совсем тривиально, второе является самым главным в конструкции натурального ряда. Поэтому говоря об определении N главный акцент делаем на индукции.

> если ты всю эту конструкцию несущую определённую информацию в областях математики и играющую важную роль в доказательствах называешь "бантиками"

Мы обсуждаем только корректное определение N, это же не повод изучать 500-страничное доказательство теоремы Ферма например?




Аноним 13/11/15 Птн 20:57:59 #410 №311285 
>>311129
помоги
Аноним 13/11/15 Птн 22:48:56 #411 №311299 
14474441365200.jpg
ЭТА НАУКА
@
ТА НАУКА
@
СПОСОБ ПРО ЭТО ДУМАТЬ
@
ЖИВЁТ
@
БЬЁТ
@
УВАЖАЕТ
Аноним 13/11/15 Птн 22:55:33 #412 №311300 
>>311130 >>311138
Ну-ка бамп.
Аноним 13/11/15 Птн 22:58:26 #413 №311301 
>>311079
Сайт не читал. На картиночках нихуя не простые числа.
Аноним 14/11/15 Суб 00:38:27 #414 №311309 
14474507079350.png
Ребят, есть тут люди, занимающиеся машинным обучением, анализом данных, мат. моделированием или чем-то в том же духе? Честно прогуливал в универе все математические дисциплины, теперь вот не знаю, куда обратиться.

Допустим, у нас есть истории клиентов, которые пользовались определённой услугой, содержащие определённые действия по дням.
Задача: построить такую модель, которая, приняв на вход данные юзера, который ещё не перестал пользоваться услугой, спрогнозировала бы, сколько он продержится.
Вопрос в том, как правильно собрать пространство признаков, объясняющих количество дней активности?
Дней много, возможных действий тоже много, так что строить таблицу со столбцами вида

день1_действиеА;день1_действиеБ;день1_действиеВ ... день47_действиеХ;день47_действиеЦ;день67_действиеЧ;

- не самый лучший выбор (но пока единственный из пришедших мне в голову), к тому же матрица получится не только растянутой, но и разряженной за счёт большого количества пользователей, которые продержались только пару дней.

Как вообще решаются такие задачи и как они правильно называются, в какую сторону копать и что гуглить?
Аноним 14/11/15 Суб 00:50:36 #415 №311311 
>>310319
БЛЯ, НЕ СЛУШАЙ ЭТИХ ДОЛБАЕБОВ. ТУТ НАДО УМНОЖИТЬ 1Ю СКОБКУ НА 3Ю, А 2Ю НА 4Ю. ЗАТЕМ ЗАМЕНА t = x^2 + 2.ПИЗДЕЦ ТУТ ДАУНЫ СИДЯТ
Аноним 14/11/15 Суб 00:51:49 #416 №311312 
>>311311
t = x^2 + 2x. ОПИСАЛСЯ
Аноним 14/11/15 Суб 06:05:37 #417 №311321 
>>311195
Можно интегрировать по пространству функций
Аноним 14/11/15 Суб 08:31:36 #418 №311327 
>>311309
Нормализация данных, feature selection.
Аноним 14/11/15 Суб 11:12:53 #419 №311334 
>>311321
Суть кратного интеграла в том, что его можно свести к повторному.
Аноним 14/11/15 Суб 11:46:09 #420 №311338 
>>311309
>Ребят, есть тут люди, занимающиеся машинным обучением, анализом данных, мат. моделированием или чем-то в том же духе?

Я иногда занимаюсь.

>Вопрос в том, как правильно собрать пространство признаков, объясняющих количество дней активности?

В общем случае никто не скажет, нужно делить модель на training и validation множества и тестировать, визуализировать, тестировать еще раз. Можно погуглить статьи с похожими подходами.
Далее, если данных у тебя много, ты можешь использовать продвинутые методы, если мало - придется очень долго заниматься manual feature selection. Чем меньше данных, тем больше дроча и шаманства. В твоем магазине данных, скорее всего, мало.

>но и разряженной за счёт большого количества пользователей, которые продержались только пару дней.

Выкинь первую колонку, а правую добей ее нулями, чтобы у каждого пользователя вектор был одной длины (большой), соответствующий состоянию пользователя в день N (без пропусков) и попробуй кластеризовать это многомерное пространство с визуализацией на 2d (через PCA, например). Возможно, тебе получится разбить пользователей на большие кластеры и работать с кластерами по-разному (типа, если юзер сделал больше 3-х действий, переводим его на другой алгоритм).

Далее, по сути, ты действие каждого пользователя можешь закодировать в виде некоторого процесса перехода из состояния в состояние (в виде или моего плоского вектора, или твоей "сжатой" таблички). Т.е. в день N у тебя пользователь находится в состоянии A, на следующий день с вероятностью P он может перейти в состояние B. А это значит, ты можешь использовать reccurent neural network, которой на вход подаются данные о новом дне (или состояние + дельта дней), а она каждый раз пытается угадать, что сделает юзер с учетом истории его действий.
Аноним 14/11/15 Суб 14:20:11 #421 №311368 
>>311309
Из всего числового ряда строишь матрицу Ганкеля, затем подсовываешь ее любому алгоритму без учителя (SOM, NG, GNG, ...). На готовую модель проецируешь новые данные и смотришь к какому кластеру она их отнесет.
Аноним 14/11/15 Суб 16:35:20 #422 №311386 
>>311334
Окей сведи мне интеграл по пространству непрерывных функций на отрезке (0,1) к повторному.
Аноним 14/11/15 Суб 17:22:47 #423 №311397 
>>311386
Я пытаюсь понять, возможно ли это.
Аноним 14/11/15 Суб 17:26:55 #424 №311399 
14475112158500.png
А вот поясните за пикрелейтед интеграл. Как его представить наглядно? D - размерность, для простоты можно считать = 1.
Аноним 14/11/15 Суб 18:07:21 #425 №311404 
>>311399
Бля ты шутишь что ли, давай ты пояснишь за обозначения, что откуда, куда. Ибо для меня E - это может быть энергия, напряженность электрического поля и.т.д
Аноним 14/11/15 Суб 18:15:47 #426 №311406 
>>311404
Это из векторного квантования. Е - ошибка квантования, P(v) - плотность вероятности распределения, v - вектор вещественнозначных чисел, wi(v) - тоже вектор вещественнозначных чисел такой же размерности что и v (скажем v - точка кластера, тогда w - центроид этого кластера), выражение (v - wi(v))^2 - это евклидово расстояние между v и w соответственно (т.е. между точкой кластера и центром этого же кластера). Т.е. по отдельности все просто и понятно, но интеграл, как в этом случае он выглядит, как его нарисовать - в душе не ебу.
Аноним 14/11/15 Суб 18:27:20 #427 №311409 
>>311406
с теорией информации не знаком, но интеграл до ужаса напоминает дисперсию случайной величины( или что тоже самое, что второй момент случайной величины)
Аноним 14/11/15 Суб 18:42:39 #428 №311413 
14475157598410.png
>>311409
>интеграл до ужаса напоминает дисперсию случайной величины
Ну да, суть примерно в том же - разброс отдельных точек кластера относительно центра в пределах плотности вероятности, только вместо матожидания случайной величины координаты центра кластера, а в качестве случайной величины - координаты элементов кластера. Но все равно непонятно, как нарисовать такой интеграл.
Аноним 14/11/15 Суб 22:02:24 #429 №311454 
>>308111 (OP)
Математики, почему логарифм единицы пл\о основанию десять равен 0?
Аноним 14/11/15 Суб 22:04:32 #430 №311455 
>>311454
лолбля. сорри. я все вспомнил.
Аноним 15/11/15 Вск 09:20:02 #431 №311573 
14475684029620.jpg
Что-то вы меня совсем запутали. Картофаны - это прикладники или «чистые».
Гротендики вроде тоже водяру любят.
Кстати, вербит же злостный бурбакист.
Аноним 15/11/15 Вск 11:52:07 #432 №311584 
>>311573
Картофаны это вторая культура.
Первая -- построение теорий, (пример -- формализм пучков, язык схем, теорема об индексе); вторая -- решение конкретных задач (пример -- комбинаторика, общая топология и функциональный анализ, cs, олимпиадная математика). Для первой теорема всегда важнее доказательства, а определение важнее теоремы, в идеальном случае там все должно вытекать из определения. Так это изложено у Гауэрса. Вербицкий называет это "core mathematics" и периферия. Гауэрс же настаивает на том, что комбинаторные результаты нередко устанавливают связь между различными ветвями и направлениями математики. Но с точки зрения Вербицкого эти направления одинаково неинтересны, поскольку относятся к "прикладной науке". И т.д.
Конкретно МГУ ближе к второй культуре, потому что там такая идеология. Для них важнейшей ценностью является количество курсов в программе, сокращать её они не готовы, и таким образом включение туда современной математики (многообразия и дифференциальные формы, когерентные пучки и классы черна, теорема римана-роха в форме гротендика) сделает невозможным освоение программы студентами. То есть, там и так студент должен знать, чем интеграл Макшейна отличается от интеграла Курцвейля-Хенстока; поскольку это соответствует специалисту широкого профиля, в понимании мехмата.
Смысл концептуального подхода в том, что он радикально упрощает математику путем введения нового, более удобного языка. См, например, формулировку уравнений Максвелла в терминах дифференциальных форм. Трудность, в этом случае, состоит только в освоении этого самого концептуального аппарата, т.е. заучивании определений и осознании их фактического содержания.
Вторая культура это поиск более элегантных доказательств, первая -- более элегантных формулировок. Вторая культура увеличивает объем математического знания, первая -- его сжимает.
Существует опасность того, что математических работ станет настолько много, что их перестанут читать, и придется передоказывать каждое утверждение десятки раз.
Вербицкий отнюдь не бурбакист. Вообще, трактат Бурбаки имеет мало общего с первокультурной математикой и создан скорее как некоторого рода формальная система, или как пример, иллюстрирующий возможность построения такой системы. Алгебраическая геометрия is nowhere near.
Вербицкий же конкретно занимается гиперкэлеровыми многообразиями и пишет учебники по метрической и римановой геометрии.
>Гротендики вроде тоже водяру любят.
Откуда ты это взял, хуй знает.
Аноним 15/11/15 Вск 12:57:54 #433 №311586 
Poconi, как побороть математическую прокрастинацию? Вроде и тема качает, и знаю что понравится если втянусь, но лень начинать вообще, что-то читать, разбираться.
Аноним 15/11/15 Вск 13:51:30 #434 №311597 
>>311584

> Алгебраическая геометрия is nowhere nea

Как же бесит эта уебанская манера школодаунов вставлять неуместные английские слова.
Аноним 15/11/15 Вск 13:52:16 #435 №311598 
>>311584

> Откуда ты это взял, хуй знает.

Читай его биографию, он же в деревне спился под конец жизни.
Аноним 15/11/15 Вск 14:19:20 #436 №311601 
>>311597
Ну так в этом вся суть выебывающихся школьников
Аноним 15/11/15 Вск 15:32:35 #437 №311605 
Последовательность {X_n} имеет предел при n -> inf. Пусть {Y_n} -- это "перемешанная" последовательность {X_n} (т.е. y_n = f(n), f: N -> X_n). Верно ли, что они тоже сходится?
Аноним 15/11/15 Вск 16:06:02 #438 №311608 
>>311584
>Вообще, трактат Бурбаки имеет мало общего с первокультурной математикой
На самом деле довольно много. Но кто читал, тот и так знает, а остальные не поймут.
Аноним 15/11/15 Вск 16:29:15 #439 №311613 
>>311605
Да, разумеется. Последовательность имеет пределом точку A - значит, в любой окрестности A лежат почти все точки последовательности. Если изменить номера точек, сами точки ведь не сдвинутся, как лежали почти все, так и будут лежать.
Аноним 15/11/15 Вск 16:40:04 #440 №311614 
>>311608
ты что у нас тут каждый школьник - первокультурщик, двигающий теорию пучков и состоящий в AMS
Аноним 15/11/15 Вск 16:49:53 #441 №311615 
>>311605
Если ты рандомно переставишь элементы и эту перестановку нельзя выразить аналитически, то нет, т.к нельзя будет указать окрестность для определенного номера(она не будет выполнять условие |x-x_n|<e)
Аноним 15/11/15 Вск 16:59:09 #442 №311618 
14475959490760.jpg
>>311338
>>311368
>>311327

Спасибо, немного упростил себе задачу и вручную разделил юзеров по активности, а в качестве целевой переменной поставил интервалы. Не так красиво, зато понятно и в целом тоже работает.

Теперь другой вопрос: как искать аномалии во временных рядах? Допустим, у нас тот же датасет в формате день_время;юзерайди;действие;сумма_оплаты;платежная_система
Ряд не стационарен и со временем показатели растут.
Задача построить такую модельку, которая бы искала вбросы по одному или нескольким признакам (допустим, в какой-то день неожиданно много оплат в ночные часы по одной из платежных систем).
Вопрос может и глупый, но я впервые сталкиваюсь с задачами прогнозирования и временными рядами, до этого препарировал только результирующие таблицы, где внутренняя динамика признака была не важна.
Что у нас тут будет объектом: юзер или день, а что его признаками? Словом, в какой вид нужно нарезать исходные данные и как такие задачит вообще принято решать?
Из понятного гугл советует только эту статью:
http://habrahabr.ru/post/251225/
Но тут в качестве данных не ряд, а та же результирующая таблица.
Буду очень благодарен за совет.
Аноним 15/11/15 Вск 17:06:58 #443 №311619 
>>311615
Лол. Перестановка последовательности - это любая биекция N->N. Достаточно "аналитически" для тебя?
Аноним 15/11/15 Вск 17:44:26 #444 №311625 
>>311613
Поддвачну этого. Можно добавить, что "почти все" означает "все кроме конечного множества точек".
Аноним 15/11/15 Вск 18:00:53 #445 №311627 
Пацаны, важно ли, чтобы кто-то проверял доказательства, которые я пишу, когда читаю учебник, или главное - это развить интуицию, а если доказательство содержит ошибку - то похуй? Просто вот я прочитал формулировку теоремы, доказал ее, потом читаю доказательство в учебнике, а оно не такое, как у меня. После этого я внимательно перепроверил свое доказательство, но все равно есть шанс, что я ошибся.

Второй вопрос: кто-то постил свои доказательства или решения задач на math.stackexchange для верификации? Как там к такому относятся? У меня с ангельским норм, потому что я погромист, но я как-то стесняюсь. Знакомых, с которыми можно скооперироваться нет, потому что я не математик и живу в Мухосранске. Если че, занимаюсь энтрилевельной математикой типа линала, матана и общей топологии.
Аноним 15/11/15 Вск 18:13:04 #446 №311629 
14476003845970.png
>>311618
>Задача построить такую модельку, которая бы искала вбросы по одному или нескольким признакам (допустим, в какой-то день неожиданно много оплат в ночные часы по одной из платежных систем).
Вот сейчас смотрю насчт временных рядов, ARIMA, автоматический подбор параметров, все дела: https://www.google.ru/url?sa=t&rct=j&q=&esrc=s&source=web&cd=1&cad=rja&uact=8&ved=0CB4QFjAAahUKEwj8vrCC2pLJAhXnnXIKHQXVDkY&url=http%3A%2F%2Fwww.jstatsoft.org%2Farticle%2Fview%2Fv027i03%2Fv27i03.pdf&usg=AFQjCNF9MBsC7olXZ1Y55K01bzD2vuzE2g&sig2=waZlm5sbv6s_gO8Vh3ZsFg хуйня полная, предсказание на 10 шагов вперед, ошибка на тестовой выборке просто запредельная:
accuracy(forecast(fit,10), a[201:209])
ME RMSE MAE MPE MAPE MASE ACF1
Training set 0.008689936 1.006038 0.782161 -0.000381481 1.303290 0.9951399 -0.002189017
Test set -1.462308612 3.207413 1.652199 -2.512794011 2.800512 2.1020849 NA
и визуально "предсказание" выглядит как говно. Временные ряды не причем, это задача идентификации модели системы, причем должна быть достаточная выборка, один выброс уровня "допустим, в какой-то день неожиданно много оплат в ночные часы по одной из платежных систем" останется незамеченным.
>>311627
Пруф ассистанты используй.
Аноним 15/11/15 Вск 19:13:57 #447 №311635 
>>311627
>Пацаны, важно ли, чтобы кто-то проверял доказательства, которые я пишу, когда читаю учебник, или главное - это развить интуицию, а если доказательство содержит ошибку - то похуй?
Главное - это получить правильную интуицию и научиться самому доводить свои доказательства до такого уровня строгости, когда ошибки весьма маловероятны. Поэтому, хотя нет ничего плохого, когда (относительно) быстро читаешь математические тексты и пропускаешь многие доказательства, придумывая общие схемы для пропущенных доказательств, и разумеется время от времени допуская ошибки, но для развития второго навыка видимо на первых порах действительно нужно чтобы доказательства проверяли.
Правда конкретно ничего не могу посоветовать - я учился в матшколе и после нее проблем с тем, чтобы при необходимости аккуратно записать доказательство у меня уже не было.
Аноним 15/11/15 Вск 19:48:40 #448 №311638 
>>311629
Про ариму и её варианты только слышал, но серьёзно не читал.
Выборка предостаточная есть, чувствительность к отклонениям, уверен, тоже можно будет настроить.
Я до сих пор не могу сообразить, как для таких задач должна выгледеть объект-признаковая матрица, ведь у нас есть и юзеры и их действия и платёжные системы и цены оплат и всё это взаимосвязано и продолжается во времени.
То есть каждое событие скаждого дня нужно проверять не только на то, выбивается ли оно из событий этого дня, но и насколько оно продолжает тенденцию предыдущих дней.

1. Что у нас тут объект, что признак, а что целевая переменная?
2. Какие инструменты (желательно, присутствующие в sklearn) могут понадобиться, чтобы чтобы решить задачу максимально просто, без претензии на запредельную точность и сложность применяемых методов?
Аноним 16/11/15 Пнд 00:59:58 #449 №311739 
Вопрос не совсем по математике, но я просто уже устал и не знаю где спросить. Как в LaTeX принято записывать остаток от деления? А именно, как записать

a = (b * c) % d

a = b \cdot c \mod{d}
или
a = (b \cdot c) \mod{d}
или как вообще принято, может как-то вообще иначе? Или киньте в меня зарубежными публикациями.
Аноним 16/11/15 Пнд 01:05:13 #450 №311740 
Я бы написал так
$a=bc \mod d$
Аноним 16/11/15 Пнд 02:10:29 #451 №311753 
Анон, посоветуй книжку про арифметику в модулях. Как решать квадратные уравнения?
Аноним 16/11/15 Пнд 09:34:33 #452 №311764 
>>311740
Ага, только у меня там всё сложнее. Произведение сложных обозначений / остаток от сложного обозначения.
Аноним 16/11/15 Пнд 09:50:47 #453 №311765 
Идите нахуй со своим машинным бучением в CS тред. Нехуя тут тему засирать.
Аноним 16/11/15 Пнд 10:58:42 #454 №311768 
>>311739 >>311740 >>311764
Сам нашёл. Буржуи почему-то пишут вот так:
>b \cdot c ≡ a \pmod{d}

И я не понимаю, почему они пишут тождество (= вообще не ставят). Я вот доказываю утверждение от противного, и мне тоже тождество ставить? Чтобы доказать. что оно неверно.
Аноним 16/11/15 Пнд 12:07:08 #455 №311779 
Поясните за топологию. На вики пишут что там ей не интересны метрические подробности. Какая там математика используется? Примеры кто то может привести? Или там одно воображение, бесмысленное и беспощадное, а математика на подсосе чтобы другому человеку можно было объснить что ты там напридумывал без размахивания руками?
Аноним 16/11/15 Пнд 12:30:16 #456 №311785 
>>311765
Но ведь это математика.
Как бы вы планарный граф в четыре цвета покрасили без комплюктера, а, матаны?
Аноним 16/11/15 Пнд 12:31:27 #457 №311787 
>>311779
Вас какая топология интересует, молодой человек?
Аноним 16/11/15 Пнд 12:39:20 #458 №311790 
>>311787
Самая что ни на есть общая, в которой пространство скрученное в бублик изучают и прочая прочая самая самая общая топология.
Аноним 16/11/15 Пнд 12:50:05 #459 №311795 
14476674053020.gif
>>311785
>>311765
Так, не разжигаем, /cs необитаема, малоизвестна и, судя по всему, вообще закрыта, а задачи машинного обучения - частный случай прикладной математики.

Я всё ещё надеюсь, что знающие люди подскажут, в каком направлении решать эту задачу >>311638 .
Аноним 16/11/15 Пнд 14:18:32 #460 №311817 
14476727127120.gif
А вот поясните за бурбакизм, например. Конкретно за пикрелейтед выражение. Как именно там получается, что вся эта хуеверть определяет пустое множество?
Аноним 16/11/15 Пнд 14:22:50 #461 №311818 
>>311817
Она не "определяет" пустое множество, а является им.
Аноним 16/11/15 Пнд 14:25:16 #462 №311819 
>>311818
Ну, пусть является. Каким образом-то, в чем суть этого выражения? В самой книге объяснения нет.
Аноним 16/11/15 Пнд 14:28:26 #463 №311821 
>>311819
Ты привык, что каждый набор символов означает какой-то объект. Здесь не так. "Значения" у этого набора символов нет. Он самобытен, самодостаточен. Пустое множество - это не "то, что обозначено набором символов, пустое множество - это и есть набор символов.

То есть, например, "собака" - это буквы с, о, б, а, к, а, записанные рядом друг с другом слева направо.
Аноним 16/11/15 Пнд 14:29:29 #464 №311822 
>>311821
>это не "то, что обозначено набором символов"
fxd
Аноним 16/11/15 Пнд 14:33:45 #465 №311823 
>>311817

Бурбакизм даже определение натурального чила не даёт, не много ли ты от него хочешь?
Аноним 16/11/15 Пнд 14:38:11 #466 №311824 
14476738910610.jpg
>>311821
Дзен какой-то. Я понимаю, что в бурбакизме буква это буква, за которой не обязано стоять что-то кроме этой буквы. Мне смысл этого выражения не понятен.
>>311823
>определение натурального чила не даёт,
Вот ты и пришел. И кстати, бурбакизм дает определение нуля и единицы, для остального достаточно аксиом Пеано.
Аноним 16/11/15 Пнд 14:44:02 #467 №311825 
>>311821
Почему именно этот набор символов?
Аноним 16/11/15 Пнд 14:49:13 #468 №311826 
>>311824

> для остального достаточно аксиом Пеано.

С точки зрения малолетнего долбоёба - безусловно.
Аноним 16/11/15 Пнд 14:52:18 #469 №311827 
>>311826
Аксиомы Пеано определяют натуральные числа. Как и https://2ch.hk/sci/src/308111/14476738910610.jpg этот пик вполне определяет тебя. Поскольку кукареканья ничего не доказывают, а "математика" есть синоним "доказательства", предлагаю тебе привести пример хоть одного натурального числа, не вписывающегося в аксиомы Пеано.
Аноним 16/11/15 Пнд 14:55:23 #470 №311828 
>>311824
Этот набор символов может быть осмыслен, но этот смысл не рассматривается бурбаками. Если бы метаматематика изучала собак, то она бы изучала правила построения текстов, содержащих слово "собака", и совсем не интересовалась бы смыслом слова "собака". Рассматривается лишь синтаксис, не семантика.

Вообще же, если интересоваться и семантикой, то философская суть такова.
Есть вещи, и есть утверждения о вещах.
И то, и другое обозначается наборами символов.
Эти наборы символов будем называть строками.

Строку, обозначающую вещь, назовём объект.
Строку, обозначающую утверждение, назовём высказывание.

Рассмотрим специальные символы, например +, - и ∈.
По смыслу, специальные символы - это либо какие-то операции, либо какие-то избранные предикаты.

Каждый специальный символ связывает два объекта.
Мы будем пользоваться прямой польской нотацией.
То есть, например, не будем писать a+b, а вместо этого будем писать + a b.

Специальные символы разделим на субстантивные и реляционные.
Субстантивные символы - это +, - и т.п.
Реляционные символы - это =, ∈ и т.п.

Если s - субстантивный символ, то s a b - объект.
Если s - реляционный символ, то s a b - высказывание.
Например, 5 и 7 - объекты.
+ 5 7 - объект.
= 5 7 - высказывание (ложное).
= + 5 7 12 - высказывание (истинное).

Кроме того, рассмотрим специальный эпсилон-оператор Гильберта (будем обозначать его буквой тау).
Суть такова.

Пусть P - какое-то высказывание.
P обозначает какое-то утверждение о вещах.
Одна из вещей, участвующих в P, может быть обозначена символом x.
τxP - это одна из вещей, которые подходят под утверждение.

Например, рассмотрим высказывание = x 7.
τx = x 7 - это объект, который обозначает вещь реального мира, которая находится в отношении, обозначенным =, с вещью, обозначенной как 7, - если такая вещь существует, в противном случае этот объект не обозначает ничего.
Например, τx= x 7 обозначает 7, потому что только 7 = 7.
В случае неоднозначности эпсилон-оператор делает выбор абсолютно произвольно.

τx ВЛАДЕЕТ (x, Сосач) обозначает Абу.
τx ЗАЕБАЛ(x, сцай) обозначает либо N-петуха, либо переводчика, но мы не знаем, кого.
τx ЖИВЁТВ(x, Россия) обозначает одного из жителей России.
Аноним 16/11/15 Пнд 15:02:15 #471 №311829 
>>311827

Лол, малолетний долбоёб пыхтел два часа над дебильной картинкой которая никому не нужна, теперь как опущенный бегает по всему сосачу со своей фотокарточкой.

Пизданись, мудило. Почитай для начало что значит дать корректное определение понятию, а потом вопросы задавай. Кстати, даже если предположить что я такой пример привести не смогу, это НИЧЕГО не докажет так как из противоречивого высказывания можно вывести все что угодно. Пуанкаре сто лет назад угорал с таких даунов как ты, почитай может поймешь чего.

Аноним 16/11/15 Пнд 15:03:07 #472 №311830 
>>311825
Конкретно тот набор символов можно, опуская двойное отрицание, переписать вот так.
∅ =def τy ∉τx∈xyy.

То есть пустое множество - это такая вещь, которая может стоять на месте y в высказывании ∉τx∈xyy.
Если от польской записи перейти к обычной скобочной, то это такой y, что (τxx∈y) ∉ y.
Символ τxx∈y обозначает одну из тех вещей, которые являются элементами y.
Если таких вещей несколько - одну из них, мы не знаем, какую.

То есть пустое множество - это такой y, что один из его элементов не является его элементом.

Аноним 16/11/15 Пнд 15:11:26 #473 №311834 
>>311779
> Какая там математика используется?
Ты имеешь в виду, что нужно знать, чтобы начать изучать топологию? Ну вот я недавно начал. По-моему для общей топологии не нужно ничего, кроме теории множеств и определенного уровня математической культуры, чтобы не охуеть.
Аноним 16/11/15 Пнд 15:11:57 #474 №311835 
Боги математики объясните простыми словами что такое многообразие, перечитал тысячи определений в различной литературы, но полного, и главное, понятного представления так и не сложилось
Аноним 16/11/15 Пнд 15:12:15 #475 №311836 
14476759354010.jpg
>>311828
>эпсилон-оператор Гильберта (будем обозначать его буквой тау).
А, ну это уже лучше. У бурбаков, кстати, про эпсилон Гильберта ничего не сказано.
>>311830
Так действительно понятнее, спс.
>>311829
>даже если предположить что я такой пример привести не смогу, это НИЧЕГО не докажет
Пок-пок, ваши пруфи не пруфи, вы все врети. Мань, это как раз однозначно докажет, что ты обосрался, причем себе же на ротешник. Ибо, если нет примеров натурального числа, не соответствующего аксиомам Пеано, то значит эти аксиомы исчерпывающе определяют N. Что в свою очередь подтверждает что твои кукареканья - это всего лишь кукареканья.
> что значит дать корректное определение понятию,
Опять же, если лично тебе что-то в математике непонятно, то это проблема твоя а не математики. Определение - это то, что исчерпывающе определяет что-то. В случае натуральных чисел определением являются аксиомы Пеано. А теперь съеби.
Аноним 16/11/15 Пнд 15:23:26 #476 №311839 
>>311836

> Ибо, если нет примеров натурального числа, не соответствующего аксиомам Пеано, то значит эти аксиомы исчерпывающе определяют N.

Ебанько, чтобы задать этот вопрос тебе нужно cначала построить аксиоматику Пеано, а ты этого сделать не можешь без индукции, которая суть N. Соснул?

> Опять же, если лично тебе что-то в математике непонятно, то это проблема твоя а не математики.

Не путай свои представления малограмотного дебила с математикой.

> Определение - это то, что исчерпывающе определяет что-то.

Аксиоматика Пеано таковой не является так как содержит порочный круг.

> В случае натуральных чисел определением являются аксиомы Пеано.

С точки зрения мудака - безусловно.
Аноним 16/11/15 Пнд 15:30:20 #477 №311841 
>>311839
>Аксиоматика Пеано таковой не является
Потому что ты так сказал? Я ж тебе предлагаю это доказать, т.е. привести пример натурального числа, не определяющегося аксиомами Пеано. Насколько я понимаю, это невозможно не только для поехавшего с подтирача, а для математики вообще. Что в свою очередь доказывает, что аксиомы Пеано определяют N. Определение же единицы есть у бурбаков.
Аноним 16/11/15 Пнд 15:38:59 #478 №311843 
Заебали. Лучше тут machine learning обсуждать, эту Санта-Барбару в виде N-петух vs. весь мир.
Аноним 16/11/15 Пнд 15:46:00 #479 №311845 
>>311843

Ты необучаемый.
Аноним 16/11/15 Пнд 15:46:14 #480 №311846 
>>311835
Сначала определим локально евклидово пространство размерности n.

Пусть M - хаусдорфово пространство.
Путь в нём есть открытые связные подмножества.
Пусть некоторые из них гомеоморфны открытым подмножествам Rn.
Пару (U, f), где U - открытое связное подмножество M, а f - гомеоморфизм на Rn, будем называть картой.
Причём f будем называть координатным отображением.

Пусть (Ui, fi) - семейство карт такое, что Ui образуют покрытие M (т.е. их объединение равно M).
Тогда такое семейство называется топологическим атласом.
M с топологическим атласом называется локально евклидовым пространством размерности n.

То есть локально евклидово пространство размерности n - это хаусдорфово пространство, каждая точка которого обладает окрестностью, гомеоморфной Rn. Топологическое многообразие - это синоним локально евклидова пространства.

Теперь определим дифференциальное многообразие.

Пусть есть две карты (U,f) и (V,g).
Так как f и g - гомеоморфизмы, у них есть обратные гомеоморфизмы f-1 и g-1.
Отображение f∘g-1 будем называть отображением склейки.
Отображение склейки - это гомеоморфизм открытых подмножеств Rn.
Значит, для отображения склейки определено понятие производной.
То есть отображения склейки могут принадлежать каким-то классам гладкости.

Пусть есть топологическое многообразие размерности n.
Пусть его атлас таков, что у любых двух карт отображение склейки относится к классу Ck.
Тогда атлас будем называть дифференциальным атласом класса Ck.
Дифференциальный атлас называется гладким, если он есть атлас класса C.
Два дифференциальных атласа одного класса называются совместными, если их объединение есть снова атлас того же класса.

Мы можем рассмотреть топологическое пространство с максимальным по включению дифференциальным атласом.
Дифференциальный атлас называется максимальным, если любой совместный с ним атлас является его подмножеством.

Базис топологического пространства - это такое семейство открытых в пространстве подмножеств, что любое открытое подмножество пространства представимо в виде объединения некоторого его подсемейства. Топологическое пространство удовлетворяет второй аксиоме счётности, если оно имеет счётный базис.

Дифференциальное многообразие размерности n класса k - это удовлетворяющее второй аксиоме счётности топологическое многообразие размерности n с зафиксированным на нём максимальным дифференциальным атласом класса Ck.

Гладкое дифференциальное многообразие - это дифференциальное многообразие с гладким атласом.
Аноним 16/11/15 Пнд 15:47:00 #481 №311847 
>>311841

Шары разуй, тебя уже тыкали рылом в доказательство.
Аноним 16/11/15 Пнд 15:47:24 #482 №311848 
>>311586
Двачую вопрос
Аноним 16/11/15 Пнд 15:50:00 #483 №311849 
>>311835
Проще говоря, топологическое многообразие - это пространство, у которого вблизи каждой точки есть координаты.
Гладкое дифференциальное многообразие - это многообразие, у которого вблизи каждой точки возможен переход к другой системе координат, причём с использованием матана - якобиан, шмакобиан, вот это всё.
Аноним 16/11/15 Пнд 15:51:55 #484 №311850 
Загоните петуха обратно под шконку, он опять весь тред засрал.
Аноним 16/11/15 Пнд 15:52:47 #485 №311851 
>>311850

Дай корректное определение натурального числа и он уйдёт. Я гарантирую это.
Аноним 16/11/15 Пнд 15:53:02 #486 №311852 
14476783827350.jpg
>>311836
>У бурбаков, кстати, про эпсилон Гильберта ничего не сказано.
Тогда тебе стоит прочитать пикрелейтед, чтобы узнать основные философские концепты. Бурбаки считали их общеизвестными.
Аноним 16/11/15 Пнд 15:57:36 #487 №311853 
>>311852

Плохая книжка, все на словах ни формальных систем нет, ни толкового изложения.
Аноним 16/11/15 Пнд 15:58:46 #488 №311854 
>>311853
Как же ты заебал.
Аноним 16/11/15 Пнд 15:59:33 #489 №311855 
>>311851
Ты имеешь ввиду не "корректное" а "устраивающее петуха"?
Потому что в данном контексте непустота пересечения этих множеств недоказана.
Аноним 16/11/15 Пнд 16:00:11 #490 №311856 
>>311855
Пробелы сам расставишь
Аноним 16/11/15 Пнд 16:01:11 #491 №311857 
>>311851>>311847
Уже сто раз написал. Докажем от противного. Допустим, аксиомы Пеано не определяют N. Тогда должно существовать как минимум одно натуральное число, не определяющееся этими аксиомами. Из того факта, что такого числа нет прямо следует, что все натуральные числа вписываются в Аксиомы Пеано, и стало быть, они однозначно определяют N.
Аноним 16/11/15 Пнд 16:01:44 #492 №311858 
>>311854

Я тебя не ебал (в рот несчитается).
Аноним 16/11/15 Пнд 16:02:57 #493 №311860 
>>311857

Ты противная дурашка, понимаешь что не можешь оперировать понятиями до их определения?
Аноним 16/11/15 Пнд 16:04:17 #494 №311861 
>>311857
Ничуть не сторонник N-петуха, но обосрался в голос от твоей сверхманёвренности.
Аноним 16/11/15 Пнд 16:04:32 #495 №311862 
>>311855

Со мной уже как минимум двое толковых математиков согласились в этом итт треде. Кроме того я приводил прямые цитаты Успенского и Степанова.
Аноним 16/11/15 Пнд 16:05:13 #496 №311863 
>>311861

На самом деле унылые у него маневры, одну и ту же хуергу переписывает сто раз. Даже кнуёбок интереснее отвечал.
Аноним 16/11/15 Пнд 16:06:06 #497 №311864 
>>311860
Еблан ты, аксиомы Пеано - это и есть определение, причем исчерпывающее. Как минимум до того момента пока ты не приведешь пример натурального числа, в них не вписывающегося. А до этого момента сасай-кудасай, придется тебе признать что я прав.
Аноним 16/11/15 Пнд 16:06:20 #498 №311865 
>>311861

Почему ты не его сторонник? Ты считаешь аксиомы Пеано являются определением N и тебя не смущает тот факт, что без индукции они даже не формулируются???
Аноним 16/11/15 Пнд 16:08:49 #499 №311866 
>>311864

Перед тем как требовать пример этого числа принеси мне построение аксиом Пеано без использования индукции на метаязыке иначе твой вопрос некорректен. Ты с тем же успехом можешь требоать посчитать 2 + 2 не дав определение что такое +.

Аноним 16/11/15 Пнд 16:09:23 #500 №311867 
>>311865
>>311865
>Ты считаешь аксиомы Пеано являются определением N
Да, считаю.
>тебя не смущает тот факт
Нет
>они даже не формулируются???
Иди нахуй.
Аноним 16/11/15 Пнд 16:11:25 #501 №311868 
>>311867

> Да, считаю.

Это характерно для малолетнего долбоёба.

> Нет

Это характерно для малолетнего долбоёба.

> Иди нахуй.

Хуй тебе в рыло.
Аноним 16/11/15 Пнд 16:14:00 #502 №311869 
>>311868
Самоутвердился? Теперь мамке похвастайся и иди уже нахуй.
Аноним 16/11/15 Пнд 16:15:09 #503 №311870 
>>>>311869

Я ебал твой мать.
Аноним 16/11/15 Пнд 16:20:54 #504 №311873 
>>311845
Знаешь, N-петух, идеологически я, может, с тобой и согласен, но мой опыт показывает, что все подобные многочасовые зануды являются как минимум психически нездоровыми, не говоря уже о самом факте засирания тредов.
Аноним 16/11/15 Пнд 16:28:50 #505 №311877 
>>311874 (OP)
>>311874 (OP)
>>311874 (OP)
>>311874 (OP)
>>311874 (OP)

Аноним 16/11/15 Пнд 16:30:46 #506 №311880 
>>311873

Идеологически это как?

PS Вообще-то это твоё машинное обучение следует признать засиранием. N намного ближе к теоретической математике.
Аноним 16/11/15 Пнд 16:36:09 #507 №311883 
>>311880
Утихомирься, или я пожалуюсь на тебя модератору.
Аноним 16/11/15 Пнд 16:40:43 #508 №311887 
>>311883

кек, мамке своей пожалуйся
Аноним 20/11/15 Птн 23:04:02 #509 №313046 
>>308457
неловко, когда листки по которым ты учился теперь на двачах
comments powered by Disqus

Отзывы и предложения